\documentclass[10pt,a4paper]{article} %format des sujets \usepackage[french]{babel} \usepackage[T1]{fontenc} \usepackage[utf8x]{inputenc} \usepackage{fancyhdr} \usepackage{amsfonts,amsmath,amssymb,mathrsfs,graphicx,amsthm,dsfont} \usepackage{euscript,mathrsfs} \usepackage{stmaryrd} \usepackage{colortbl} \usepackage{comment} \usepackage{tikz} \usepackage{mathdots} \setlength{\topmargin}{-25mm} \setlength{\textheight}{25.5cm} \setlength{\oddsidemargin}{-10mm} \setlength{\textwidth}{17.5cm} \newcommand{\htrait}{\smallskip\hspace{-0.6cm}\rule{20cm}{0.5pt}\smallskip} % Pour séparer les exos \newcount\exocount % compteur exo \exocount=0 \newcount\espcount % compteur qsp \espcount=0 \newcount\planchecount \planchecount=0 \newcommand{\exoe}{\vspace{3mm}\par\advance\planchecount by1 \begin{center} \Large {\bf SUJET E\the\planchecount}\\ \vspace*{1cm} {\large {\bf Exercice principal S\the\planchecount}} \end{center} \vspace*{2cm}} \newcommand{\Espe}{\vspace{3mm} \begin{center} \Large {\bf Exercice sans préparation E\the\planchecount}\\ \end{center} \vspace*{2cm}} \newcommand{\exo}{\vspace{3mm}\par\advance\exocount by1 \noindent{\hspace{-0.4cm}\bf Exercice principal n\textsuperscript{o}\the\exocount\ \ \\ \ \\ }} \newcommand{\Esp}{\vspace{3mm}\par\advance\espcount by1 \noindent{\hspace{-0.4cm}\bf Exercice sans préparation n\textsuperscript{o}\the\espcount\ \ \\ \ \\}} \everymath{\displaystyle} % applique un displaystyle à toutes les commandes de math %ensembles de nombres \def\C{\mathbb{C}} \def\D{\mathbb{D}} \def\K{\mathbb{K}} \def\H{\mathbb{H}} \def\N{\mathbb{N}} \def\Z{\mathbb{Z}} \def\Q{\mathbb{Q}} \def\R{\mathbb{R}} \def\U{\mathbb{U}} \def\E{\mathbb{E}} \def\P{\mathbb{P}} \def\V{\mathbb{V}} \def \A{{\cal A}} \newcommand{\Nstar}{\ensuremath{\mathbb{N}^{*}}} \newcommand{\Zstar}{\ensuremath{\mathbb{Z}^{*}}} \newcommand{\Qstar}{\ensuremath{\mathbb{Q}^{*}}} \newcommand{\Rstar}{\ensuremath{\mathbb{R}^{*}}} \newcommand{\Cstar}{\ensuremath{\mathbb{C}^{*}}} \newcommand{\Rplus}{\ensuremath{\mathbb{R}_{+}}} \newcommand{\Rmoins}{\ensuremath{\mathbb{R}_{-}}} \newcommand{\Rplusstar}{\ensuremath{\mathbb{R}_{+}^*}} \newcommand{\Rmoinsstar}{\ensuremath{\mathbb{R}_{-}^*}} \newcommand{\Rplusetoile}{\ensuremath{\mathbb{R}_{+}^*}} \newcommand{\Rmoinsetoile}{\ensuremath{\mathbb{R}_{-}^*}} % Divers \newcommand{\e}{\mathrm{e}} %e de exponentielle \newcommand{\Id}{\hbox{\rm{Id}\,}} \newcommand{\1}[1]{\mathds{1}_{#1}}% la fonction indicatrice pour les proba \newcommand{\llb}{\llbracket} \newcommand{\rrb}{\rrbracket} %Relation de comparaison \renewcommand{\leq}{\leqslant} \renewcommand{\geq}{\geqslant} \newcommand{\peto}[1]{\rm{o}\left(#1\right)} \newcommand{\gdo}[1]{\rm{O}\left(#1\right)} \newcommand{\eq}[1]{\operatorname*{\sim}_{#1} } %Algèbre \def \M{{\cal M}} \def \L {{\cal L}} \newcommand{\Sp}{\text{Sp}\,} \newcommand{\Ker}{\mbox{ Ker }} \newcommand{\im}{\mbox{ Im }} \newcommand{\Vect}{\ensuremath{\mathop{\rm Vect\,}\nolimits}} \newcommand{\rg}{\ensuremath{\mathop{\rm rg\,}\nolimits}} \newcommand{\tr}{\ensuremath{\mathop{\rm Tr\,}\nolimits}} \newcommand{\abs}[1]{\left| #1 \right|}%valeur absolue \newcommand{\norme}[1]{\left|\left| #1 \right|\right|}%norme \newcommand{\prodscal}[2]{\langle #1 ; \; #2 \rangle} %produit scalaire %Mettre entre .. \newcommand{\pa}[1]{\ensuremath{\left(#1\right)}}%parenthèses \newcommand{\paf}[2]{\ensuremath{\left(\frac{#1}{#2}\right)}} % fractions entre parenthèses \newcommand{\acco}[1]{\ensuremath{\left\{ #1\right\}}}%accolades \newcommand{\crocint}[1]{\left[\begin{array}{c} #1 \end{array}\right]}% grands crochets pour les IPP % Intégrations \newcommand{\dx}{\ensuremath{\,\mathrm{d}x}} \newcommand{\dt}{\ensuremath{\,\mathrm{d}t}} \newcommand{\du}{\ensuremath{\,\mathrm{d}u}} \newcommand{\dv}{\ensuremath{\,\mathrm{d}v}} \newcommand{\dy}{\ensuremath{\,\mathrm{d}y}} \newcommand{\parmi}[2]{\left( \begin{array}{c} #2 \\ #1 \end{array} \right)} %algo de Gauss \newcommand{\opgauss}[1]{\stackrel{\mbox{\scriptsize$\begin{array}{l}#1\end{array}$}}{\longrightarrow}}% Pour décrire les opérations de pivot arg= un tableau du type L_1\devient & L-1+... \newcommand{\devient}{\leftarrow} \newcommand{\echange}{\leftrightarrow} % Matrices par blocs \newcommand{\bigzero}{\mbox{\normalfont\Large\bfseries 0}} \newcommand{\rvline}{\hspace*{-\arraycolsep}\vline\hspace*{-\arraycolsep}} % Numérotation des question \newcommand{\be}{\begin{enumerate}} \newcommand{\ee}{\end{enumerate}} \newcommand{\bi}{\begin{itemize}} \newcommand{\ei}{\end{itemize}} % Fonction \newcommand{\fonc}[4]{ \begin{array}{lcl} {#1} & \rightarrow & {#2} \\ {#3} & \mapsto & {#4} \\ \end{array} } \renewenvironment{comment}{ \htrait\vspace*{4mm}\qquad \textbf{Solution :}\\ }{\par} % Pour masquer ou afficher les solutions \pagestyle{empty} \begin{document} \title{Rapport et exercices oraux HEC Mathématiques(E) } \author{Juin-Juillet 2021} \date{} \maketitle Le bilan de la session 2021 de l'oral de mathématique en filière ECE est satisfaisant. Le niveau des candidats est très hétérogène : les notes se sont étalées entre 3 et 20. La moyenne s'établit à 11,47 et l'écart-type à 4,42. Les candidats les plus faibles ont montr\'{e} d'importantes lacunes de cours, de grosses faiblesses en calcul, et une absence totale de réactivité aux indications du jury. \'A contrario, le jury a pu apprécier des prestations tout à fait remarquables de candidats réfléchis, précis et efficaces. Le jury aimerait insister sur les points suivants auprès des futur.e.s candidat.e.s et de leurs enseignant.e.s. \bi \item Les raisonnements graphiques et les tracés de courbes sont des compétences que nous souhaiterions fortement valoriser à l'avenir. Nous avons été surpris de voir de nombreux candidats esquiver les questions de tracé de graphes issus d'études de fonction ne présentant pas de difficulté technique. Malgré l'aide du jury, certains candidats ont insisté pour sauter ces questions et passer à autre chose. C'est tout à fait contre-productif. Que les futurs candidats aient bien conscience que l'an prochain, ces questions seront incontournables. \item Le programme est à connaître dans son intégralité. Par exemple, la loi faible des grands nombres (telle qu'elle est énoncée dans le programme officiel) n'est pas connue de certains candidats. De la même manière, lorsque les candidats ont eu comme question de cours le théorème de la bijection, il a souvent été difficile de savoir ce que signifiait le mot "bijection" pour les candidats. \item L'usage des quantificateurs est parfois difficile pour les candidats : par exemple la définition précise d'une valeur propre est souvent compliquée à obtenir. \item Les questions informatiques ne doivent pas être négligées par les candidats. Elles ont permis à certains candidats d'améliorer significativement leur note. \item L'oral n'est pas un deuxième écrit : lors de la présentation de l'exercice préparé, certains candidats recopient leur brouillon dans leur intégralité, même sur des questions routinières. D'autres, au contraire lisent leurs notes sans laisser de trace écrite : une juste équilibre doit être trouvé pour permettre au jury de suivre précisément ce qu'à fait le candidat sans perdre de temps inutilement. \item Tout n'est pas joué à l'issue de la préparation : au cours de la présentation de l'exercice préparé, le jury pose des questions pour aiguiller le candidat vers la solution. Il convient d'y être attentif. Il est souvent utile d'écrire au tableau ce qui est proposé par le jury. Par ailleurs, une prestation peut être jugée excellente sans que le candidats ne traite beaucoup de questions, alors qu'un candidat traitant de manière approximative un grand nombre de questions en maltraitant au passage les théorèmes de son cours risque d'être déçu par sa note finale. \item La question sans préparation est aussi très importante. Là encore, pour la plupart d'entre elles, le candidat peut tout à fait faire bonne impression sans aller au bout de la question. L'important est de réfléchir et d'écouter les indications du jury. Le candidat n'est pas obligé de parler instantanément en découvrant l'énoncé. Le jury a pu constater que certains candidats, qui avaient complètement raté l'exercice préparé ont redressé la barre sur l'exercice sans préparation, et parfois dans les toutes dernières minutes. \ei Voici quelques sujets proposés cette année. Nous publions aussi leurs corrigés, mais insistons sur le fait que ces corrigés sont indicatifs, ont été écrit à l'intention des membres de jury et ne correspondent pas toujours exactement à un attendu. \newpage \exoe Soient $a$ et $b$ deux r\'eels tels que $a>0$ et la fonction $$f_{a,b}: \quad \fonc{\R}{\R}{x} { \left\{ \begin{array}{cc} \frac{x+a-b}{a^2} & \mbox{ si } b-a\le x \le b \\ \\ \frac{-x+a+b}{ a^2}& \mbox{ si } b0$ et $\displaystyle \int_0^a\big(F(x)\big)^n dx\ge 0$, $\E(T_n)r$, $e'_k$ est associé à la valeur propre 0. Déterminer une base de $\im(g)$ et montrer que $\Ker(g)\cap \im(g)=\acco{0_{\R^n}}$ \item Peut-on dire que si $\Ker(g)\cap \im(g)=\acco{0_{\R^n}}$ alors $g$ est diagonalisable ? \ee\ee \begin{comment} \be \item Programme ECE2 page 8. Un endomorphisme de $\R^3$ est diagonalisable si et seulement si \fbox{la somme des dimensions de ses sous-espaces propres vaut 3.} \item $A=-4I+J$\\ $A^2=J^2-8J+16I$ or $J^2=3J$ donc :\\ \fbox{$A^2=-5J+16I=-5A-4I$}\\ On en déduit que $A\ensuremath{\left(-\frac{1}{4}A-\frac{5}{4}I\right)}=I$ donc $A$ est inversible et \fbox{$A^{-1}=-\frac{1}{4}A-\frac{5}{4}I$} \item Si $x$ est un vecteur propre associé à $\lambda\neq 0$, $h(x)=\lambda x$ et donc $x=\frac{1}{\lambda}h(x)\in \im(h)$ \fbox{Si $x$ est un vecteur propre associé à $\lambda\neq 0$, $x\in \im(h)$} \item \be\item $\im(f)=$vect($(f(e_1),f(e_2),f(e_3))$ \fbox{$\im(f))=\Vect(u_1)$ où $u_1=(1,1,1)$ et $(u_1)$ est une base de $\im{f}$.} Soit $x=(x_1,x_2,x_3)\in\R^3$ $x \in \ker{f} \Leftrightarrow f(x)=0 \Leftrightarrow x_1+x_2+x_3=0$\\ D'après le théorème du rang, $\dim{\ker{f}}=2$.\\ Une base de $\ker{(f)}$ est donc \fbox{$(u_2,u_3)$ avec $u_2=(-1,1,0)$ et $u_3=(-1,0,1)$} \item $\dim{(\ker{(f)})}=2$ donc 0 est valeur propre de $f$ et la dimension du sous-espace propre associé est 2.\\ Le vecteur $u_1$ vérifie $f(u_1)=3u_1$ et est donc un vecteur propre de $f$ associé à la valeur propre 3. La famille $(u_1,u_2,u_3)$ est donc libre (concaténation de bases de sous-espaces propres) La somme des dimensions des sous-espaces propres de $f$ vaut 3 donc $f$ est diagonalisable, \fbox{donc $J$ est diagonalisable.}\\ \fbox{$\Ker(f)\neq \acco{0}$, donc $J$ n'est pas inversible.} \item Comme dim($\im(f))=1$, alors dim($(\im(f)\cap \Ker(f))\in\acco{0;1}$ Or, si dim($(\im(f)\cap \Ker(f))=1$ alors $\im(f)\cap \Ker(f)=\im(f)$ et donc $f(u_1)=0$. Ce n'est pas le cas. \fbox{$(\im(f)\cap \Ker(f))=\acco{0_{\R^3}}$} \ee \item \be Soit X un vecteur propre de J, il existe un réel $\lambda \in \acco{0;3}$ tel que $JX=\lambda X$.\\ Or $A=-4I+J$ donc $AX=(-4I+J)X=-4X+\lambda X=(-4+\lambda)X$\\ Comme $X$ est non nul \fbox{$X$ est bien un vecteur propre de A.}\\ $J$ est diagonalisable donc il existe une base de $M_{3,1}(\R)$ formée de vecteurs propres de $J$, qui sont aussi vecteurs propres de $A$. \fbox{Donc $A$ est diagonalisable.}\\ Ses valeurs propres sont $-4+3=-1$ et $-4+0=-4$. \ee \item \be\item $\im(g)=\Vect(g(e'_1),...g(e'_n))=\Vect(\lambda_1 e'_1,\cdots,\lambda_r e'_r,0,..0))=\Vect(e'_1,\cdots,e'_r)$ Mais alors si $x\in \im(g)\cap \Ker(g)$, il existe $x_1,..x_r\in\R$ tels que : $x=\sum_{k=1}^r x_k e'_k$ et $g(x)=\sum_{k=1}^r x_k f(e'_k)=\sum_{k=1}^r\lambda_kx_k e'_k=0.$ Ainsi pour tout $k\in[|1,r|]$, $\lambda_kx_k=0$ et comme $\lambda_k\neq 0$, $x_k=0$. \fbox{Si $g$ est diagonalisable, alors $\Ker(g)\cap \im(g)=\acco{0_{\R^n}}$} \item \fbox{Non}, il suffit de prendre un endomorphisme non diagonalisable tel que $\Ker(g)=\acco{0_{\R^n}}$, par exemple l'endomorphisme induit par $\begin{pmatrix}1&1\\0&1\end{pmatrix}$ \ee \ee \end{comment} \newpage \Espe Soit $(X_n)_{n\in\N}$ une suite de variables aléatoires définies sur l'espace probabilisé $(\Omega, \mathscr{T},\mathbb{P})$. \begin{enumerate} \item On suppose dans cette question que \bi \item $\forall n\in \N $ $ X_n(\Omega)\subset \N$ \item $ \forall n\in \N , $ $X_n$ admet une espérance et $\lim \mathbb{E}(X_n )= 0$. \ei La suite de variables aléatoires $(X_n)$ converge-t-elle en loi? \item On suppose maintenant \bi \item $\forall n\in \N , $ $ X_n $ admet une espérance et $\lim \mathbb{E}(X_n)= 0$ \ei La suite de variables aléatoires $(X_n)$ converge-t-elle en loi? \item On suppose dans cette question que \bi \item $ \forall n\in \N $ $ X_n(\Omega)\subset\R^{+}$ \item $ \forall n\in \N , $ $X_n$ admet une espérance et $\lim \mathbb{E}(X_n)= 0$. \ei La suite de variables aléatoires $(X_n)$ converge-t-elle en loi? \end{enumerate} \begin{comment} \begin{enumerate} \item D'après l'inégalité de Markov $\mathbb{P}\left(X_n\geqslant \frac{1}{2}\right)\leqslant 2E(X_n)$.\\ Donc par encadrement $\lim\limits_{n\to +\infty} \mathbb{P}\left(X_n\geqslant \frac{1}{2}\right)=0$.\\ En considérant l'événement contraire, $\lim\limits_{n\to +\infty} \mathbb{P}\left(X_n< \frac{1}{2}\right)=1$.\\ Or $X_n$ ne prend que des valeurs entières. Donc $\mathbb{P}\left(X_n\geqslant \frac{1}{2}\right)=\mathbb{P}(X_n=0)$.\\ Donc $\lim\limits_{n\to +\infty} \mathbb{P}\left(X_n=0\right)=1$.\\ Ainsi \fbox{$X_n$ converge en loi vers la variable aléatoire certaine égale à $0$. } \item Pour tout entier $n$, on note $X_n$ la variable aléatoire telle que : $X_n(\Omega)=\{-n;n\}$ et $\mathbb{P}(X_n=-n)=\mathbb{P}(X_n=n)=\frac{1}{2}$.\\ Alors $X_n$ admet une espérance et $\mathbb{E}(X_n)=0$.\\ Mais $\forall x\in \mathbb{R},\: \exists n_0\in\mathbb{N},\: \forall n\geqslant n_0,\ -n\leqslant xx)\leqslant \mathbb{P}(X_n>x)$. Donc $\mathbb{P}(X_n> x)\leqslant \frac{E(X_n)}{x}$.\\ Par encadrement $\lim\limits_{n\to +\infty} \mathbb{P}(X_n> x)=0$.\\ En passant à l'événement contraire, $\lim\limits_{n\to+\infty} \mathbb{P}(X_n\leqslant x)=1$.\\ Soit $x$ un réel strictement négatif.\\ Pour tout entier $n$, comme $X_n$ est à valeurs dans $\R_+$, $\mathbb{P}(X_n\leqslant x)=0$. Ainsi $\lim\limits_{n\to+\infty} F_{X_n}(x)=\begin{cases} 0 & \textrm{si } x<0\\ 1 & \textrm{si }x>0\end{cases}$.\\ Or cette dernière fonction est la fonction de répartition d'une variable aléatoire certaine égale à $0$. \fbox{Nécessairement, $(X_n)$ converge en loi} \end{enumerate} \end{comment} \newpage \exoe Soit $(X_n)_{n\in\N}$ une suite de variables aléatoires définies sur un même espace probabilisé $(\Omega,\mathscr{T},\mathbb{P})$, mutuellement indépendantes suivant toutes une loi de Bernoulli de paramètre $p$ où $p\in]0,1[$. On pose pour tout entier $n$ non nul, $Y_n=X_{n-1}X_n$ et $Z_n=Y_{n}Y_{n+1}$. \begin{enumerate} \item%1 {\bf Question de cours:} \'Enoncer l'inégalité de Bienaymé-Tchebychev. \item %2 Montrer que pour tout entier $n$ non nul, $Y_n$ suit une loi de Bernoulli de paramètre à déterminer. \item %3 Pour tout entier $n$ supérieur ou égal à $3$, les variables aléatoires $Y_1,\ldots, Y_n$ sont-elles deux à deux indépendantes, mutuellement indépendantes ? \item %4 Pour tout entier $n$ non nul, déterminer la loi de $Z_n$. \item%5 Pour tout entier naturel $n$ non nul, on pose $T_n=\frac{1}{n}\sum\limits_{k=1}^n Y_k$. \'Ecrire une fonction Scilab d'entrée $(n,p)$ et donnant en sortie une simulation de la variable aléatoire $T_n$. \item %6 \begin{enumerate} \item Montrer que pour tout entier $n$ non nul, $T_n$ admet une espérance et la déterminer. \item Montrer par récurrence que pour tout entier $n$ supérieur ou égal à $2$, $$\mathbb{E}\left(\left(\sum\limits_{k=1}^n Y_k\right)^2\right)=\sum\limits_{k=1}^n\mathbb{E}(Y_k^2)+2\sum\limits_{k=1}^{n-1}\mathbb{E}(Z_k)+(n-1)(n-2)p^4.$$ \item En déduire que $T_n$ admet une variance et la déterminer. \end{enumerate} \item %7 Montrer que $T_n$ est un estimateur convergent de $p^2$. \end{enumerate} \begin{comment} \begin{enumerate} \item%1 Programme officiel ECE2 p17. Soit $X$ une variable aléatoire admettant une espérance et une variance.\\ Alors \fbox{$\forall \varepsilon>0,\: \mathbb{P}\left(\abs{X-\mathbb{E}(X)}\geqslant \varepsilon\right)\leqslant \frac{\mathbb{V}(X)}{\varepsilon^2}$.} \item%2 Soit $n$ un entier naturel non nul.\\ Alors $Y_n(\Omega)=\{0,1\}$. Et $\mathbb{P}(Y_n=1)=\mathbb{P}\big((X_{n-1}=1)\cap(X_n=1\big)$.\\ Or $X_n$ et $X_{n-1}$ sont indépendantes. Donc $\mathbb{P}(Y_n=1)=\mathbb{P}\big(X_{n-1}=1\big)\mathbb{P}\big(X_n=1\big)=p^2$.\\ Ainsi \fbox{$Y_n$ suit une loi de Bernoulli de paramètre $p^2$.} \item%3 Soit $k$ et $\ell$ deux entiers distincts tels que $k<\ell$.\\ Si $k<\ell-1$, alors comme $X_{k-1}$, $X_k$, $X_{\ell-1}$ et $X_{\ell}$ sont des variables aléatoires deux à deux indépendantes. Donc $Y_{k}$ et $Y_{\ell}$ sont indépendantes.\\ Si $k=\ell-1$, alors $\mathbb{P}\big((Y_{k}=1)\cap(Y_{\ell}=1)\big)=\mathbb{P}\big((X_{k-1}=1)\cap(X_k=1)\cap(X_{k+1}=1)\big)=p^3$ car $p^4-p^3=p^3(1-p)$ et $p\in]0,1[$ .\\ Or $\mathbb{P}(Y_k=1)\mathbb{P}(Y_{\ell}=1)=p^4$. \fbox{Donc $Y_k$ et $Y_{\ell}$ ne sont pas indépendantes.}\\ Donc les variables aléatoires $Y_1$, $Y_2$, $\cdots$, $Y_n$ ne sont pas mutuellement indépendantes. \item%4 Soit $n$ un entier naturel non nul.\\ Alors $Z_n(\Omega)=\{0,1\}$. Et $\mathbb{P}(Z_n=1)=\mathbb{P}\big((Y_{n+1}=1)\cap(Y_n=1)\big)=p^3$ \fbox{$Z_n\hookrightarrow{\mathcal B}(p^3)$}. \item %7 \begin{verbatim} function Tn=T(n,p) X=grand(n+1,1,"bin",1,p) Tn=0 for i=1:n Tn=Tn+X(i,1)*X(i+1,1) end; Tn=Tn/n; endfunction \end{verbatim} \item %5 \begin{enumerate} \item %a Les variables aléatoires $Y_1$, $Y_2$, $\cdots$, $Y_n$ ont même loi et admettent une espérance $p^2$. Donc $T_n$ admet une espérance.\\ Et \fbox{$\mathbb{E}(T_n)=\frac{1}{n}\sum\limits_{k=1}^np^2=p^2$} \item %b \begin{description} \item[Initialisation] Si $n=2$, alors \begin{eqnarray*} \mathbb{E}\left(\left(\sum\limits_{k=1}^2Y_k\right)^2\right) & = & \mathbb{E}\left(Y_1^2+Y_2^2+2Y_1Y_2\right)\\ & = & \mathbb{E}(Y_1^2)+\mathbb{E}(Y_2^2)+2\mathbb{E}(Z_1)\\ & = & \sum\limits_{k=1}^2\mathbb{E}(Y_k)+2\sum\limits_{k=1}^{2-1}\mathbb{E}(Z_k)+(2-1)(2-2)p^4 \end{eqnarray*} \item[Hérédité] Soit $n$ un entier supérieur ou égal à $2$.\\ Supposons que $\mathbb{E}\left(\left(\sum\limits_{k=1}^n Y_k\right)^2\right)=\sum\limits_{k=1}^n\mathbb{E}(Y_k^2)+2\sum\limits_{k=1}^{n-1}\mathbb{E}(Z_k)+(n-1)(n-2)p^4$. \begin{eqnarray*} \textrm{Alors } \mathbb{E}\left(\left(\sum\limits_{k=1}^{n+1} Y_k\right)^2\right)& = & \mathbb{E}\left(\left(\sum\limits_{k=1}^n Y_k\right)^2+Y_{n+1}^2+2Y_{n+1}\sum\limits_{k=1}^nY_k\right)\\ & = & \mathbb{E}\left(\left(\sum\limits_{k=1}^n Y_k\right)^2\right)+\mathbb{E}(Y_{n+1}^2)+2\mathbb{E}\left(Y_{n+1}\sum\limits_{k=1}^nY_k\right)\\ & = & \mathbb{E}\left(\left(\sum\limits_{k=1}^n Y_k\right)^2\right)+\mathbb{E}(Y_{n+1}^2)+2\mathbb{E}\left(Y_{n+1}\sum\limits_{k=1}^nY_k\right)\\ & \underset{HR}{=} & \sum\limits_{k=1}^n\mathbb{E}(Y_k^2)+2\sum\limits_{k=1}^{n-1}\mathbb{E}(Z_k)+(n-1)(n-2)p^4+\mathbb{E}(Y_{n+1}^2)+2\sum\limits_{k=1}^n\mathbb{E}(Y_{n+1}Y_k)\\ & = & \sum\limits_{k=1}^{n+1}\mathbb{E}(Y_k^2)+2\sum\limits_{k=1}^{n}\mathbb{E}(Z_k)+2\sum\limits_{k=1}^{n-1}\mathbb{E}(Y_{n+1}Y_k)+(n-1)(n-2)p^4\\ & = & \sum\limits_{k=1}^{n+1}\mathbb{E}(Y_k^2)+2\sum\limits_{k=1}^{n}\mathbb{E}(Z_k)+2(n-1)p^4+(n-1)(n-2)p^4\\ & = & \sum\limits_{k=1}^{n+1}\mathbb{E}(Y_k^2)+2\sum\limits_{k=1}^{n}\mathbb{E}(Z_k)+n(n-1)p^4 \end{eqnarray*} \end{description} Ainsi pour tout entier $n$ supérieur ou égal à $2$, \fbox{$\mathbb{E}\left(\left(\sum\limits_{k=1}^n Y_k\right)^2\right)=\sum\limits_{k=1}^n\mathbb{E}(Y_k^2)+2\sum\limits_{k=1}^{n-1}\mathbb{E}(Z_k)+(n-1)(n-2)p^4.$} \item %c Comme $T_n$ prend un nombre fini de valeurs, $T_n$ admet une variance. \begin{eqnarray*} \mathbb{V}(T_n)& = & \mathbb{E}(T_n^2)-\left(\mathbb{E}(T_n)\right)^2\\ & = & \frac{1}{n^2}\mathbb{E}\left(\left(\sum\limits_{k=1}^nY_k\right)^2\right)-p^4\\ & = & \frac{1}{n^2} \left(\sum\limits_{k=1}^n\mathbb{E}(Y_k^2)+2\sum\limits_{k=1}^{n-1}\mathbb{E}(Z_k)+(n-1)(n-2)p^4\right)-p^4\\ & = & \frac{1}{n^2} \left(\sum\limits_{k=1}^np^2+2\sum\limits_{k=1}^{n-1}p^3+(n-1)(n-2)p^4\right)-p^4\\ & = & \frac{p^2}{n}+\frac{2(n-1)}{n^2}p^3+\frac{(n-1)(n-2)-n^2}{n^2}p^4\\ & = & \frac{p^2}{n}+\frac{2(n-1)}{n^2}p^3+\frac{2-3n}{n^2}p^4 \end{eqnarray*} \end{enumerate} \item %6 D'après l'inégalité de Bienaymé Tchebychev appliqué à $T_n$, $\mathbb{P}\left(\abs{T_n-\mathbb{E}(T_n)}\geqslant \varepsilon\right)\leqslant \frac{\mathbb{V}(T_n)}{\varepsilon^2}$. Or d'après la question précédente $\lim\limits_{n\to +\infty} \mathbb{V}(T_n)=0$. Donc par encadrement $\lim\limits_{n\to +\infty }\mathbb{P}\left(\abs{T_n-p^2}\geqslant \varepsilon\right)=0$. \\ Ainsi \fbox{$T_n$ est un estimateur convergent de $p^2$.} \end{enumerate} \end{comment} \newpage\Espe Soit $n$ un entier naturel non nul et $E={\cal M}_n(\R) $ l'ensemble des matrices carr\'ees d'ordre $n$ \`a coefficients r\'eels. Soit $B$ et $C$ deux \'el\'ements de $E$ tels que $B^2 = C^2 =I$ o\`u $I$ est la matrice identit\'e. On pose $A=BC$. \be\item Montrer que la matrice $A$ est inversible. Quelle est son inverse? \item Montrer que $A$ est semblable à $A^{-1}$. \item Soit $X \in {\cal M}_{n,1}(\R) $ une matrice colonne. Montrer que si $X$ est vecteur propre de $A$, il l'est aussi de $A^{-1}$. \ee \begin{comment} \be\item $B^2=C^2=I$ donc $B$ et $C$ sont inversibles et $B^{-1}=B$ et $C^{-1}=C$. \fbox{Donc $BC$ est inversible et $A^{-1}= C^{-1}B^{-1}= CB$.} \item $A=BC=CCBC=CA^{-1}C=C^{-1}A^{-1}C$ donc \fbox{$A$ et $A^{-1}$ sont semblables.} \item Si $X$ est vecteur propre, $X\neq 0$ et $\exists \lambda\in\R\mbox { tel que } AX = \lambda X$ donc $X=\lambda A^{-1}X$ Et comme $A$ est inversible, $\lambda \neq 0$ donc \quad $A^{-1}X = {1\over \lambda} X$, et comme $X\neq 0$ \fbox{ X est un vecteur propre de $A^{-1}$} \item Question suppl\'ementaire \'eventuelle: Montrer que $BX$ est aussi vecteur propre de $A$. $ABX=BCBX=B(CB)X=BA^{-1}X=B({1\over \lambda}X) = {1\over \lambda}BX $ $(\lambda\neq 0)$. De plus BX$\neq 0$ car B est inversible \fbox{$BX$ est aussi vecteur propre de $A$.} \ee \end{comment} \newpage \exoe \be\item {\bf Question de cours}: énoncés des inégalités des accroissements finis. \bigskip Soit $f$ une fonction continue sur $\R_{+}$. On définit la fonction $$g:\left\{\fonc{\R_{+}^{*}}{\R}{x}{\frac{1}{x} \int_{x^{2}}^{x^{4}} f(t) \dt}\right.$$ \item Pour quelles valeurs du réel $x$ a-t-on $x^{2}\leq x^{4}$ ? \item \be\item Montrer que $g$ est continue sur $\R_{+}^{*}$. Que vaut $g(1) $ ? \item Montrer qu'il existe $M\in\Rplus$ tel que $\forall x\in ]0;1]$, $\abs{g(x)}\leq M\abs{x^3-x}$. \item Montrer que $g$ est prolongeable par continuité en $0 .$ On appelle encore $g$ la fonction prolongée. Préciser la valeur de $g(0)$. \ee \item Montrer que $g$ est dérivable sur $\R_{+}^*,$ donner $g^{\prime}(x)$ pour tout $x$ de $\R_{+}^{*}$. On admet provisoirement que $g$ est dérivable en 0 et que $g^{\prime}(0)=-f(0)$. \item Applications: \be\item Soit $f$ telle que $\forall x \in \R_{+} \quad f(x)=1$. Vérifier les résultats obtenus ci-dessus et tracer la courbe représentative de $g$. \item Soit $f$ une densité de probabilité, nulle sur $\R_{-}^{*}$, continue et strictement positive sur $\R_{+}$. \be\item \'Etudier le signe de $g$ et $\lim _{x \rightarrow+\infty} g(x)$. \item On admet de plus que $g'$ s'annule seulement en deux valeurs $\alpha$ et $\beta$ sur $\Rplusstar$ Tracer l'allure de la courbe représentative de $g$. \ee \ee \item On va maintenant prouver que $g$ est dérivable en 0. On note $F(x)=\int_0^xf(t)\dt$ pour $x\in\Rplus$. \be\item \'Ecrire la formule de Taylor-Young à l'ordre 1 pour $F$ en 0. \item En déduire des équivalents quand $x$ tend vers 0 de $F(x^4)$ et de $F(x^2)$ et conclure. \ee\ee \begin{comment} \be\item Programme officiel ECE1, page 14. Soit $f$ une fonction dérivable sur un intervalle I. Si $\forall x\in I$, $m\leq f'(x)\leq M$ alors, pour $a,b\in I$ vérifiant $a\leq b$, $m(b-a)\leq f(b)-f(a)\leq M(b-a)$ Si $\forall x\in I$, $\abs{ f'(x)}\leq k$ alors, pour $a,b\in I$, $\abs{ f(b)-f(a)}\leq k\abs{b-a}$ \item $x^4-x^2=x^2(x^2-1)=x^2(x-1)(x+1)$ D'où le tableau de signe $\begin{array}{|c|cccccccccc|} \hline x&-\infty&&-1&&0&&1&&+\infty&\\\hline x^4-x^2&&+&0&-&0&-&0&+&&\\\hline \end{array}$ \item \be\item Soit $F$ une primitive de $f \operatorname{sur} \R_{+} .$ F est de classe ${\mathcal C}^1$. Alors $g(x)=\frac{F\left(x^{4}\right)-F\left(x^{2}\right)}{x}$. Donc \fbox{$g$ est continue sur $\R_{+}^{*}$} (quotient de fonctions continues sur $R_{+}^{*}$, le dénominateur ne s'annulant pas.) De plus $g(1)=0$ \item Si $x \in] 0,1]$, $x^4,x^2\in[0;1]$ $F^{\prime}=f$ et $f$ est continue sur $[0,1],$ qui est un intervalle fermé borné. Donc $|f|$ est bornée sur [0,1]. Ainsi $\exists M \geq 0$ tel que $: \quad \forall x \in[0,1] \quad|f(x)| \leq M$ Donc avec l'inégalité des accroissements finis $\forall x \in]0,1] \quad\left|F(x^{4})-F(x^{2})\right| \leq M\abs{x^4-x^2}$. Et comme $x>0$ : \fbox{$|g(x)| \leq M\left|x^{3}-x\right|$} \item Et par encadrement : $\quad \lim _{x \rightarrow 0} g(x)=0 .$ \fbox{Conclusion: $g$ est prolongeable par continuité en $0 .$ Et $g(0)=0$.} \ee \item $F$ est dérivable (c'est une primitive) donc $g$ est dérivable sur $\R_{+}^{*}$ et $\forall x \in \R_{+}^{*}$ \fbox{$ g^{\prime}(x)=\frac{-1}{x} g(x)+\frac {1}{x}\left(4 x^{3} f\left(x^{4}\right)-2 x f\left(x^{2}\right)\right)$} \item \be\item $g(x)=x^{3}-x ; \quad g^{\prime}(0)=-1=-f(0) ; \quad g(1)=0 ; \quad g^{\prime}(1)=2 \quad$ et $\quad g$ est convexe sur $\R_{+}$ $g'(x)=3x^2-1$, on a un minimum en $x_0=\frac{1}{\sqrt{3}}$, $g(x_0)=\frac{1}{3\sqrt3}-\frac1{\sqrt3}=\frac{-2}{3\sqrt{3}}$ \begin{tikzpicture}[scale=1] \clip (-1,-1) rectangle (3,3); \draw[thick, domain=0:5.8, samples=80] plot (\x,{\x*\x*\x-\x}) ; \draw [dashed](-1,0)--(3,0) (0,-1)--(0,3); \draw (0,0) node{$\bullet$}; \draw(1,0) node{$\bullet$}; \draw (1,0) node[below right]{$1$} node{$|$}; \draw (0,0) node[below right]{$0$} node{$|$}; \draw (0,1) node[below right]{$1$} node{$-$}; \end{tikzpicture} \item \be\item Soit $F$ la fonction de répartition d'une variable aléatoire de densité $f$. $F$ est une primitive de $f$ sur $\R_{+}$ $g(x)=\frac{1}{x}\left(F\left(x^{4}\right)-F\left(x^{2}\right)\right)$ $F$ est strictement croissante sur $\R_{+}$ d'où $g(x) \leq 0$ si $x \in[0,1] \quad$ et $\quad g(x) \geq 0$ si $x \geq 1$ $\lim _{+\infty} F=1$ donc \fbox{$\lim _{x \rightarrow+\infty} g(x)=0$} \item $g(0)=0 ; \quad g^{\prime}(0)=-f(0)<0 ; \quad g(1)=0 ; \quad g^{\prime}(1)=2 f(1)>0$ Comme $g$ est dérivable sur $\R_{+}$ et $g$ admet un seul minimum local et un seul maximum local sur $\R_{+}^{*}$ Nécessairement on doit avoir $\alpha\in]{0};{1}[$ et $\beta\in]1;{+\infty}[$ Preuve rigoureuse (si ils ont l'intuition, commencer par leur laisser tracer la courbe) -> le minimum de $g$ sur $[0;1]$ est strictement négatif car $g'(0)<0$ et $g(0)=0$, il est donc dans $]0;1[$ et il vaut $\alpha$ -> On doit avoir $\beta>1$, car sinon, $g'$ étant continue, on aurait $g'$ de signe constant sur $]1;{+\infty}[$ et $g$ serait strictement croissante sur $]1;{+\infty}[$ ce qui est incompatible avec les valeurs trouvées. D'où l'allure de la courbe. \begin{tikzpicture}[scale=4] \clip (-0.3,-0.4) rectangle (3,0.3); \draw[thick, domain=0.01:5.8, samples=80] plot (\x,{((exp(-\x*\x)-exp(-\x*\x*\x*\x))/\x}) ; \draw [dashed](-1,0)--(3,0) (0,-1)--(0,3); \draw (0,0) node{$\bullet$}; \draw(1,0) node{$\bullet$}; \draw (1,0) node[below right]{$1$} node{$|$}; \draw (0,0) node[below right]{$0$} node{$|$}; \draw (0,1) node[below right]{$1$} node{$-$}; \end{tikzpicture} \ee\ee \item \be\item D'après la formule de Taylor-Young appliqué à $F$ au voisinage de 0 ($F$ est bien ${\mathcal C^1}$ sur $\Rplus$): $F(x)=F(0)+f(0)x+o(x)$ quand $x$ tend vers 0. \item Donc $F(x)\eq{0^+}f(0)x$ (car $f(0)\neq 0$) et $F(x^2)\eq{0^+} f(0)x^2$ et $F(x^4)\eq{0} f(0^+)x^4$ Or, $\frac{g(x)-g(0)}{x-0}=\frac{F(x^4)-F(x^2)}{x^2}=\frac{F(x^4)}{x^2}-\frac{F(x^2)}{x^2}$ Le premier terme tend vers 0, le deuxième vers $-f(0)$ \fbox{$g$ est dérivable en 0 et $g(0)=-f'(0)$} \ee \ee \end{comment} \newpage\Espe $$g:\quad \fonc{\R}{\R}{t}{ \left \{ \begin{array}{cc} \frac{9\ln(3)}{ 3^{t}} & \mbox{ si } t\in \left[ 2;+\infty \right[\\ 0 & \mbox{ sinon} \end{array} \right .} $$ On admet que $g$ est une densit\'{e} de probabilit\'{e}. On note $Y$ une variable al\'{e}atoire admettant $g$ comme densit\'{e}. \be \item On note $Z$ la variable al\'{e}atoire \'{e}gale \`{a} la partie enti% \`{e}re de $Y$. On rappelle que la partie enti\`{e}re d'un nombre r\'{e}el $x $ est le plus grand entier inf\'{e}rieur ou \'{e}gal \`{a} $x$.\\ D\'{e}terminer la loi de probabilit\'{e} de $Z$. \item On note $U$ la variable définie par $Y-Z$. Déterminer sa fonction de répartition. \ee \begin{comment} \be\item Comme $P\left( Y\ge 2\right) =1$ alors $Z\left( \Omega \right) =\Nstar \setminus \acco{1} $ Pour tout entier $k\geq2 :\left( Z=k\right) =\left( k\le Y0$.\\ On définit la fonction $$f:\fonc{D}{\R}{x}{\ln\left(\e^x-\e^{-x}\right) }$$ On note $(C)$ sa courbe repr\'esentative dans un rep\`ere orthonorm\'e $(O,\vec{\imath},\vec{\jmath})$.\\ \item \be \item Sans chercher à le calculer, prouver l'existence d'un unique réel $\alpha$ vérifiant $ f(\alpha)=0$ et montrer que $\alpha<1.$ \item Compléter le programme SCILAB suivant permettant d'avoir une valeur approchée par défaut de $\alpha$ à $0.01$ près. \begin{verbatim} a= 0; b= 1; while if then b=(a+b)/2; else a=(a+b)/2; end; end; disp(a) \end{verbatim} \item Calculer la valeur explicite de $\alpha$. \ee \item \be \item Donner la position relative de la droite $(\Delta)$ d'équation $y=x$ par rapport à $(C)$. \item Calculer $\lim_{x\rightarrow +\infty} \left(f(x)-x\right) $ et tracer sur un même graphe la courbe $(C)$ et la droite $(\Delta)$ \ee \item Soit $\lambda$ un réel, on note $g_{\lambda}$ la fonction d\'efinie par~: $$g_{\lambda}\;:\quad\fonc{\R}{\R}{x}{ \begin{cases} 0 & \text{si } x<\alpha \\ \frac{\lambda}{\e^{2x}-1} & \text{si }x\geq \alpha \end{cases}}$$ \be \item On pose $h\;:\;x\mapsto f(x)-x$. Apr\`es avoir calculé $h^{\prime }(x)$, déterminer $\lambda$ en fonction de $\alpha$ pour que $g_{\lambda}$ soit une densit\'e de probabilit\'e d'une certaine variable al\'eatoire $X$. \item Donner la fonction de r\'epartition $G_{\lambda}$ de $X$. \ee \item Montrer qu'au voisinage de 0, $f(x)$ est équivalent à $\ln(x)$. \ee \begin{comment} \be \item Programme officiel ECE1 page 12. Toute fonction continue et strictement monotone sur un intervalle $I$ défi nit une bijection de $I$ sur l'intervalle $f(I)$. \item $\e^x-\e^{-x}>0 \Leftrightarrow \e^{2x}-1>0 \Leftrightarrow \e^{2x}>1 \Leftrightarrow x>0$. Donc \fbox{$D=\Rplusstar$.} \item \be \item $f$ est d\'{e}rivable sur $\Rplusstar$ et : $\forall x \in \Rplusstar, f^{\prime }\left( x\right) =\frac{\e^{x}+\e^{-x}}{\e^{x}-\e^{-x}}$\\ $f^{\prime }$ est strictement positive sur $\Rplusstar$ donc $f$ est strictement croissante sur $\Rplusstar$. $\lim_{x \to 0}\ln \left( \e^{x}-\e^{-x}\right) = -\infty $ et $\lim_{x \to +\infty}\ln \left( \e^{x}-\e^{-x}\right)=+\infty$. $f$ étant continue, d'après le théorème de la bijection, l'équation $f(x)=0$ admet une unique solution $\alpha$ sur $\Rplusstar$. De plus $f(1)=\ln(e-e^{-1})>0$ (en effet $e>2$, $e^{-1}<1$, donc $e-e^{-1}>1$ et donc $f(1)>0$. Comme $f$ est strictement croissante, \fbox{$\alpha<1$.} \item On reconnaît une dichotomie. \begin{verbatim} a=0; b=1; while b-a>0.01 if log(exp((a+b)/2)-exp(-(a+b)/2))>0 then b=(a+b)/2; else a=(a+b)/2; end; end; disp(a) \end{verbatim} \item $f(\alpha)=0 \Leftrightarrow \ln \left(\e^{x}-\e^{-x}\right) =0\Leftrightarrow e^{x}-e^{-x}=1\Leftrightarrow e^{2x}-1=e^{x} \Leftrightarrow e^{2x}-e^{x}-1=0$\\ Soit $X=\e^{x}.$ L'\'{e}quation $X^{2}-X-1=0$ du second degr\'{e} a pour discriminant 5 et pour racines : $\frac{1-\sqrt{5}}{2}<0$ et $\frac{1+\sqrt{5}}{2}>0$. Donc l'unique solution est \fbox{$\alpha =\ln \left( \frac{1+\sqrt{5}}{2}\right) $} \ee \item \be \item $f\left( x\right) -x=\ln \left( \e^{x}\left( 1-\e^{-2x}\right) \right) -\ln(\e^x)= \ln \left( 1-e^{-2x}\right) <0$ \fbox{la courbe de $f$ est en dessous de la droite $(\Delta)$.} \item \fbox{$f\left( x\right) -x=\ln \left( 1-\e^{-2x}\right)\rightarrow 0$} \item $\;\;$ \begin{tikzpicture}[scale=0.5] \clip (-1,-2) rectangle (2.3,2.3); \draw[thick, domain=-1:2, samples=80] plot (\x,{\x}) ; \draw[domain=0.001:2, samples=80] plot (\x,{ln(exp(\x)-exp(-\x))}) ; \draw [dashed](-1,0)--(4,0) (0,-2)--(0,4); \draw (-1,0)--(0,0); \draw (0,0) node{$\bullet$}; \draw (0,1) node[below left]{$1$} node{$-$}; \draw (1,0) node[below right]{$1$} node{$|$}; \end{tikzpicture} {\it Le mot asymptote n'est pas dans le programme} \ee \item \be \item $h$ est d\'{e}rivable sur $D$ et $h^{\prime }\left( x\right) =\frac{\e^{x}+\e^{-x}}{\e^{x}-\e^{-x}}-1=\frac{2\e^{-x}% }{\e^{x}-\e^{-x}}=\frac{2}{\e^{2x}-1}=\frac2{\lambda}g_{\lambda}(x)$ Pour $\lambda >0$ on a $g_{\lambda }\ge 0$ et continue sur $\Bbb{R-\left\{ \alpha \right\} }$ $\int_{-\infty }^{+\infty }g_{\lambda}(x)\dx$ est impropre en $-\infty $ et en $+\infty :$ \begin{itemize} \item En $-\infty :$ $\int_{-\infty }^{\alpha }g=\int_{-\infty }^{\alpha}0=0$ \item En $+\infty :\int_{\alpha }^{M}g_{\lambda}\left( t\right) dt=\left[ \frac{\lambda }{2}% h\left( x\right) \right] _{\alpha }^{M}=\frac{\lambda }{2}\left( h\left(M\right) -h\left( \alpha \right) \right) $ Or $h\left( \alpha \right) =f\left( \alpha \right) -\alpha =-\alpha $ et $% f\left( M\right) -M\rightarrow 0$ quand $M\rightarrow +\infty $ Donc $\int_{\alpha }^{M}g\left( t\right) dt\rightarrow \frac{\lambda \alpha }{2}$ \item Donc $\int_{-\infty }^{+\infty }g$ converge et vaut $\frac{\lambda \alpha }{2}$ \end{itemize} \fbox{Donc $g_{\lambda }$ est une densit\'{e} si et seulement si $\lambda =2/\alpha $}. \item \fbox{On a $G_{\lambda }\left( t\right) =0$ si $t\le \alpha $ et $% \displaystyle G_{\lambda }\left( t\right) =\frac{1}{\alpha }\left( h\left( t\right) +\alpha \right) $ si $t\ge \alpha $.} \ee \item On fait un DL à l'ordre 2, $e^x-e^{-x}=2x+o(x)$ est équivalent à $2x$ Donc $f(x)=\ln\pa{x\times \frac{e^x-e^{-x}}{x}}=\ln(x)+\ln\paf{e^x-e^{-x}}{x}$ Le premier terme tend vers l'infini, le deuxième vers $\ln(2)$, le premier est prépondérant. \fbox{$f(x)$ est équivalent à $\ln(x)$ quand $x$ tend vers 0} \ee \end{comment} \newpage\Espe Soit $\lambda$ un r\'eel strictement positif. Soit $X$ une variable al\'eatoire suivant une loi exponentielle de param\'etre $\lambda$, définie sur un espace probabilis\'e ($\Omega,{\mathcal A},\P)$. On note $V=\max(X,1-X)$ le maximum de $X$ et de $1-X$, et l'on admet que V est bien une variable aléatoire sur ($\Omega,{\mathcal A},\P)$. \begin{enumerate} \item Déterminer $V(\Omega)$, l'ensemble des valeurs possibles pour la variable aléatoire $V$. \item D\'eterminer la fonction de r\'epartition de $V$. V est-elle une variable aléatoire à densité ? \end{enumerate} \begin{comment} \begin{enumerate}\item Si $x\geq (1-x)$ ssi $x\geq \frac{1}2$.\\ Si $X(\omega)\geq \frac{1}{2}$, alors $V(\omega)=X(\omega)$, donc $\left[\frac{1}{2};+\infty\right[\subset V(\Omega)$ Si $X(\omega)\leq \frac{1}{2}$, alors $V(\omega)=1-X(\omega)$. Or, si l'on note $g:x\mapsto 1-x$, l'image de $]0;\frac12]$ par $g$ vaut $\left]\frac12;1\right[$ \fbox{$V(\Omega)=\left[\frac{1}{2};+\infty\right[\cup \left]\frac12;1\right[=\left[\frac{1}{2};+\infty\right[$} \item Attention, les variables $X$ et $1-X$ ne sont pas ind\'ependantes. Si $v\leq \frac12$, $F_V(v)=0$, et si $v>\frac12$, $\P(V\leq v)=\P(X\leq v\cap1-X\leq v)=\P(X\leq v\cap X\geq 1-v)$ Ainsi $F_V(v)=\P(1-v\leq X\leq v)$ (en effet, comme $v\geq \frac12$, on a bien $1-v\leq v$) Ainsi $F_V(v)=F_X(v)-F_X(1-v)$. Et il faut faire attention. A-t-on $1-v>0$ ? Si $v\in[\dfrac{1}{2};1]$, $F_V(v)=\exp(\lambda(1-v))-\exp(-\lambda v)$. Mais si $v>1$, $F_V(v)=1-\exp(-\lambda v)$. $F_V$ est bien continue sur $\R$ (en particulier en $\frac12$ et en 1, et ${\mathcal C}^1$ sur $\R$ privé de $\frac12$ et de $1$.) \fbox{V est une variable à densité} \ee \end{comment} \newpage \exoe Le jeu de mémory est composé de $n$ ($n$ étant un entier naturel non nul) paires d'images deux à deux distinctes, sur une seule des $n$ paires sont représentés des chatons. Ces images sont réparties en deux tas: chaque paire aura une de ses images dans chaque tas. Les images sont posées face cachée. A chaque étape, une carte de chaque tas est retournée. Si les deux cartes retournées forment la paire de chatons, alors le jeu s'arrête, sinon les cartes sont retournées et les tas à nouveau mélangés. \\ Deux joueurs $A$ et $B$ jouent en parallèle. Ils possèdent chacun leur propre jeu de mémory et jouent indépendamment, mais réalisent leurs étapes en même temps. Soit $X$ (respectivement $Y$) le nombre d'étapes de jeu effectuées par le joueur $A$ (respectivement $B$) lorsqu'il trouve la paire de chatons. Soit $M=\textrm{max}(X,Y)$. On admet que $M$ est une variable aléatoire. \\ \begin{enumerate} \item%1 {\bf Question de cours:} \'Enoncer la définition de l'espérance d'une variable aléatoire discrète. \item %2 Donner la loi de $X$, son espérance et sa variance \item %3 Pour tout entier naturel $k$, déterminer $\mathbb{P}(M\leqslant k)$. \item%4 Montrer que la série $\sum\limits_{k\geqslant 0}\mathbb{P}(M>k)$ converge. \item%5 Montrer que pour tout entier naturel $K$ non nul, $\sum\limits_{k=1}^{K}k\mathbb{P}(M=k)=-K\mathbb{P}(M>K)+\sum\limits_{k=0}^{K-1}\mathbb{P}(M>k)$. \item%6 En déduire que $M$ admet une espérance. \item%7 Montrer que la suite $\big(K\:\mathbb{P}(M>K)\big)_{K\geqslant 0}$ converge vers $0$. \item%8 Déterminer $\mathbb{E}(M)$. \end{enumerate} \begin{comment} \begin{enumerate} \item %1 Programme officiel ECE1 page 19. On dit que la variable aléatoire discrète $X$ telle que $X(\Omega)=\{x_n,n\in \mathbb{N}\}$ admet une espérance quand la série \fbox{$\sum\limits_{n\geqslant 0}x_n\mathbb{P}(X=x_n)$ converge absolument.}\\ En cas de convergence, on appelle espérance de $X$ le réel $\mathbb{E}(X)=\sum\limits_{n= 0}^{+\infty}x_n\mathbb{P}(X=x_n)$. \item%2 \fbox{$X$ suit une loi géométrique de paramètre $\frac{1}{n^2}$. $\mathbb{E}(X)=n^2$ et $\mathbb{V}(X_n)=n^2(n^2-1)$.} \item %3 Soit $k$ un entier naturel.\\ Remarquons que $(M\leqslant k)=(X\leqslant k)\cap (Y\leqslant k)$.\\ Par indépendance des variables aléatoires, $\mathbb{P}(M\leqslant k)=\mathbb{P}(X\leqslant k)\mathbb{P} (Y\leqslant k)$.\\ Or $X$ et $Y$ ont même loi. Donc $\mathbb{P}(M\leqslant k)=\mathbb{P}(X\leqslant k)^2$.\\ Or $\mathbb{P}(X\leqslant k)=\sum\limits_{i=1}^k\mathbb{P}(X=i)=\sum\limits_{i=1}^k\frac{1}{n^2}\left(1-\frac{1}{n^2}\right)^{i-1}=1-\left(1-\frac{1}{n^2}\right)^k$.\\ Ainsi \fbox{$\mathbb{P}(M\leqslant k)=\left(1-\left(1-\frac{1}{n^2}\right)^k\right)^2$.} \item%4 D'après la question précédente pour tout entier naturel $k$, $\mathbb{P}(M>k)=2\left(1-\frac{1}{n^2}\right)^k-\left(1-\frac{1}{n^2}\right)^{2k}$.\\ Or $\left(1-\frac{1}{n^2}\right)\in[0,1[$ et $\left(1-\frac{1}{n^2}\right)^2\in[0,1[$.\\ Donc les séries géométriques $\sum\limits_{k\geqslant 0} \left(1-\frac{1}{n^2}\right)^k$ et $\sum\limits_{k\geqslant 0} \left(1-\frac{1}{n^2}\right)^{2k}$ convergent.\\ Ainsi par combinaison linéaire, la série \fbox{$\sum\limits_{k\geqslant 0}\mathbb{P}(M>k)$ converge. } \item%5 Soit $K$ un entier naturel non nul.\\ Rappelons que pour tout entier naturel $k$ non nul, $\mathbb{P}(M=k)=\mathbb{P}(M>k-1)-\mathbb{P}(M>k)$. \begin{eqnarray*} \sum\limits_{k=1}^Kk\mathbb{P}(M=k) & = & \sum\limits_{k=1}^Kk\left(\mathbb{P}(M>k-1)-\mathbb{P}(M>k)\right)\\ & = & \sum\limits_{k=1}^Kk\Big(\mathbb{P}(M>k-1)-\sum\limits_{k=1}^Kk\mathbb{P}(M>k)\Big)\\ & = & \sum\limits_{k=1}^Kk\ \mathbb{P}(M>k-1)-\sum\limits_{k=1}^Kk\ \mathbb{P}(M>k)\\ & = & \sum\limits_{k=0}^{K-1}(k+1)\mathbb{P}(M>k)-\sum\limits_{k=1}^Kk\mathbb{P}(M>k)\\ & = & \boxed{\sum\limits_{k=0}^{K-1}\mathbb{P}(M>k)-K\mathbb{P}(M>K)} \end{eqnarray*} \item %6 Pour tout entier naturel $K$ non nul, $K\ \mathbb{P}(M>K)\geqslant 0$.\\ Donc $\forall K\in\N^*,\:\sum\limits_{k=1}^Kk\ \mathbb{P}(M=k)\leqslant \sum\limits_{k=0}^{K-1}\mathbb{P}(M>k)$.\\ Or la série $\sum\limits_{k\geqslant 0}\mathbb{P}(M>k)$ est une série convergente à termes positifs.\\ Donc $\forall K\in\N^*,\: \sum\limits_{k=0}^{K-1}\mathbb{P}(M>k)\leqslant \sum\limits_{k=0}^{+\infty}\mathbb{P}(M>k)$.\\ Donc $\forall K\in\N^*,\:\sum\limits_{k=1}^Kk\ \mathbb{P}(M=k)\leqslant \sum\limits_{k=0}^{+\infty}\mathbb{P}(M>k)$.\\ Ainsi la suite des sommes partielles de la série à termes positifs $\sum\limits_{k\geqslant 1}k\ \mathbb{P}(M=k)$ est majorée. Donc la série $\sum\limits_{k\geqslant 1}k\ \mathbb{P}(M=k)$ converge absolument (car les termes sont positifs.).\\ Ainsi \fbox{$M$ admet une espérance.} \item %7 Soit $K$ un entier naturel non nul.\\ $\mathbb{P}(M>K)=\sum\limits_{k=K}^{+\infty} \mathbb{P}(M=k)$.\\ Or $\forall k\in\N,k\geqslant K,\: K\mathbb{P}(M=k)\leqslant k\mathbb{P}(M=k)$.\\ Comme $X$ admet une espérance, la série $\sum\limits_{k\geqslant 0} \mathbb{P}(M=k)$ converge.\\ Donc $0\leqslant K\sum\limits_{k=K}^{+\infty}\mathbb{P}(X=k)\leqslant \sum\limits_{k=K}^{+\infty}k\mathbb{P}(M=k)$.\\ Comme reste d'une série convergente, $\lim\limits_{K\to +\infty} \sum\limits_{k=K}^{+\infty}k\mathbb{P}(M=k)=0$.\\ Donc par encadrement \fbox{$\lim\limits_{K\to +\infty} K\mathbb{P}(M>K)=0$.} \item %7 D'après les questions précédentes \begin{eqnarray*} \mathbb{E}(M) & = & \sum\limits_{k=0}^{+\infty}\mathbb{P}(M>k)\\ & = & 2\sum\limits_{k=0}^{+\infty} \left(1-\frac{1}{n^2}\right)^k-\sum\limits_{k=0}^{+\infty} \left(1-\frac{1}{n^2}\right)^{2k}\\ & = & \frac{2}{1-\left(1-\frac{1}{n^2}\right)}-\frac{1}{1-\left(1-\frac{1}{n^2}\right)^2}\\ &=& 2n^2 -\frac{n^4}{2n^2-1}\\ & = &\boxed{ n^2\frac{3n^2-2}{2n^2-1}} \end{eqnarray*} \end{enumerate} \end{comment} \newpage \Espe \noindent Soit $n$ un entier naturel supérieur ou égal à 2. Soit $A$ une matrice de $\M_n(\R)$. On note $^t\!A$ la transpos\'ee de $A$ et $O_n$ la matrice nulle de $\M_n(\R)$. \\ On suppose $^t\!AA=A^t\!A$ et qu'il existe un entier $p\in \N^*$ tel que $A^p=O_n.$ \begin{enumerate} \item Montrer que $^t\!AA=O_n$. \item Que peut-on en d\'eduire concernant la matrice $A$ ?\\ \end{enumerate} \begin{comment} \begin{enumerate} \item La matrice $^t\!AA$ est sym\'etrique. En tant que matrice sym\'etrique r\'eelle, $^t\!AA$ est diagonalisable (programme officiel de ECE2, page 8). Ainsi, il existe une matrice inversible $P$ telle que $$^t\!AA=P\,\mbox{diag}(\lambda_1,\dots,\lambda_n)P^{-1}.$$ Comme $A$ et sa transpos\'ee commutent, $(^tAA)^p=(^tA)^p A^p=O_n$. Ainsi, $$O_n=(^t\!AA)^p=P\,\mbox{diag}(\lambda_1^p,\dots,\lambda_n^p)P^{-1}.$$ On en d\'eduit que chaque $\lambda_i$ est nul, puis que \fbox{$^t\!AA=O_n$.} \item On note $a_{ij}$ le coefficient g\'en\'erique de $A$ et $b_{ij}$ le coefficient g\'en\'erique de $B={}^t\!AA$. Par la formule du produit matriciel, pour tout $i\in [\![1,n]\!]$, $$0=b_{ii}=\sum_{k=1}^n a_{ik}^2$$ Il suit que pour tout couple $(i,k)\in [\![1,n]\!]^2$, $a_{ik}=0$. \fbox{La matrice $A$ est donc la matrice nulle.}\\ \end{enumerate} \noindent \textbf{Question suppl\'ementaire \'eventuelle.} D\'eterminer l'ensemble des matrices $A\in \M_n(\R)$ telles que $$A^t\!A A^t\!AA=I_n.$$ \noindent \textbf{R\'eponse.} On commence par remarquer que $A$ est inversible avec $A^{-1}={}^t\!A A^t\!AA.$\\ Donc $A^{-1}$ est sym\'etrique r\'eelle, et par suite $A$ l'est aussi. Ainsi, $A$ est diagonalisable et v\'erifie $A^5=I_n$. On trouve alors que la seule matrice solution est \fbox{$A=I_n$.} \end{comment} \newpage \exoe Dans tout l'exercice, toutes les variables al\'eatoires sont d\'efinies sur un m\^eme espace probabilis\'e $(\Omega, {\cal A},P)$. Soient $m$ un réel et $$ f_{m}: \fonc{\R}{\R}{x}{\frac{e^{-x+m}}{\left(1+e^{-x+m}\right)^{2}}} $$ \be \item{\bf{\bf Question de cours}}: convergence en loi d'une suite de variables aléatoires. \item On pose $f=f_{0}$. Vérifier que $f$ est paire et tracer sa courbe représentative. Comment en déduire la courbe représentative de $f_{m}$ pour $m$ réel ? \item Vérifier que $f_{m}$ est une densité de probabilité. \item On note, pour $m\in\R$, $X_{m}$ une variable aléatoire de densité $f_{m} .$ \be\item Soit $m\in\R$, quelle est la fonction de répartition de $X_{m}$ ? \item La suite $(X_n)_{n\in\N}$ converge-t-elle en loi ? \ee \item Soit $m\in\R$ fixé. Montrer que $X_{m}$ admet une espérance et donner sa valeur. On note maintenant, pour $m\in\R$, $Y_{m}=\ln \left(1+e^{X_{m}}\right).$ \item\be \item Quelle est la loi de la variable aléatoire $Y_{0}$ ? \item La suite $(Y_{\frac{1}{n}})_{n\in\N}$ converge-t-elle en loi ? \ee \ee \begin{comment} \be\item Programme officiel ECE2 page 17. Une suite $(X_n)_{n\in\Nstar}$ de variables aléatoires converge en loi vers $X$ si $\lim _{n \rightarrow+\infty}F_{X_n}(x)=F_X(x)$ en tout réel $x$ où $F_X$ est continue. $f$ est densité de probabilité ssi $f$ est continue sur $\R$ éventuellement privé d'un nombre fini de points, $f$ est positive et $\int_{-\infty}^{+\infty}f(t)\dt$ converge et vaut 1. \item $f$ est continue, dérivable sur $\R$. $\forall x \in \R \quad f(-x)=\frac{e^{x}}{\left(1+e^{x}\right)^{2}}=\frac{e^{-2 x} e^{x}}{e^{-2 x}\left(1+e^{x}\right)^{2}}=f(x) \quad$ donc $f$ est paire. La dérivée de $u \mapsto \frac{u}{(u+1)^{2}} \quad$ est $\quad u \mapsto \frac{1-u}{(1+u)^{3}}$. Donc $\forall x \in \R \quad f^{\prime}(x)=\frac{-e^{-x}\left(1-e^{-x}\right)}{\left(1+e^{-x}\right)^{3}}$ donc $f$ est strictement décroissante sur $\R_{+}$. $\lim _{+\infty} f=0 \quad$ et $\quad f(0)=\frac{1}{4}$ Comme $\forall x \in \R \quad f_{m}(x)=f(x-m)$, la courbe de $f_{m}$ se déduit de celle de $f$ par la translation de vecteur $m \vec{i}$ \begin{tikzpicture}[scale=2] \clip (-3,-0.5) rectangle (4,1); \draw[thick, domain=-3:4, samples=80] plot (\x,{exp(-\x)/((1+exp(-\x))^2)}) ; \draw[dashed, domain=-3:4, samples=80] plot (\x,{exp(-(\x-1))/((1+exp(-\x+1))^2)}); \draw [dashed](-3,0)--(4,0) (0,-1)--(0,1); \draw(0,1) node[below right]{$1$} node{$-$}; \draw (1,0) node[below right]{$1$} node{$|$}; \draw (0,0) node[below right]{$0$} node{$|$}; \end{tikzpicture} \item Vu que $\forall x \in \R \quad f_{m}(x)=f(x-m),$ il suffit de vérifier que $f$ est une densité de probabilité. $f$ est continue, positive sur $\R$. $$ \forall A \in \R_{+} \int_{0}^{A} f(x) d x=\left[\frac{-1}{1+e^{-x}}\right]_{0}^{A}=\frac{1}{2}-\frac{1}{1+e^{-A}} $$ $\lim _{A \rightarrow+\infty} \int_{0}^{A} f(x) d x=\frac{1}{2},$ donc $\int_{-\infty}^{+\infty} f(x) d x$ converge et $\int_{-\infty}^{+\infty} f(x) d x=1$ \fbox{$f$ est donc bien une densité de probabilité.} \item \be\item Soit $F_{\mathrm{m}}$ la fonction de répartition de $X_{m}$. $\forall x \in \R \quad $\fbox{$F_{m}(x)=\frac{1}{1+e^{-x+m}}$.} \item Pour $x$ fixé, $\lim_{x\rightarrow +\infty}F_n(x)=0$. Mais la fonction nulle n'est pas la fonction de répartition d'une variable aléatoire \fbox{La suite $(X_m)$ ne converge pas en loi.} \ee \item On remarque $X_{m}=X_{0}+m .$ On étudie l'espérance de $X_{0}$. $x \mapsto x f(x)$ est continue sur $\R,$ positive sur $\R_{+}$ $\lim _{x \rightarrow+\infty} x^{3} f(x)=0$ donc $x f(x)=\circ\left(\frac{1}{x^{2}}\right)$ en $+\infty$ et comme $\int_{1}^{+\infty} \frac{\dx}{x^{2}}$ converge, $\int_{1}^{+\infty} x f(x) \dx$ converge Comme $f$ est paire, $\int_{-\infty}^{+\infty} x f(x) d x$ converge et $E\left(X_{0}\right)=0 .$ \fbox{D'où: $\quad E\left(X_{m}\right)=m$.} \item \be\item $Y_{0}$ prend ses valeurs dans $\R_{+}^{*}$. Soit $F_{Y_{0}}$ la fonction de répartition de $Y_{0}$. $\begin{array}{ll}\forall y \leq 0 & F_{Y_{0}}(y)=0 \\ \forall y>0 & F_{Y_{0}}(y)=\P\left(\ln \left(1+e^{X_{0}}\right) \leq y\right)=\P\left(e^{X_{0}} \leq e^{y}-1\right)=\P\left(X_{0} \leq \ln \left(e^{y}-1\right)\right)\end{array}$ $\forall y>0 \quad F_{Y_{0}}(y)=\frac{1}{1+e^{-\ln \left(e^{y}-1\right)}}=\frac{1}{1+\frac{1}{e^y-1}}=1-e^{-y}$ \fbox{ $Y_{0}$ suit la loi exponentielle de paramètre $1 .$} \item De même : $F_{Y_{\frac{1}{n}}}(y)=0$ si $y\leq 0$ Si $y>0$, $F_{Y_{\frac{1}{n}}}(y)=\P(X_\frac{1}{n}\leq \ln(e^y-1))=\frac{1}{1+e^{-\ln(e^y-1)+\frac{1}{n}}}=$\fbox{$=\frac{e^y-1}{e^\frac{1}{n}+e^y-1}$} Donc pour $y\leq 0$, $\lim_{n\rightarrow +\infty}F_n(y)=0$, pour $y> 0$, $\lim_{n\rightarrow +\infty} F_{Y_{\frac{1}{n}}}(y)=1-e^{-y}=F_{Y_{0}}(y)$ \fbox{La suite $(Y_{\frac{1}{n}})$ converge en loi vers $Y_{0}$} \ee\ee \end{comment} \newpage \Espe On définit la suite $(u_n)$ pour tout entier naturel $n$ par $u_n=\frac{n^3+6n^2-5n-2}{n!}$.\\ \be \item Montrer que la série de terme général $u_n$ converge et calculer sa somme. (On pourra remarquer que $(X(X-1)(X-2),X(X-1),X,1)$ est une base de $\R_3[X]$.) \item Montrer que $\lim_{n\rightarrow +\infty}u_n=0$ et écrire un programme SCILAB qui pour un réel $\varepsilon>0$ donné par l'utilisateur propose une valeur de $n$ tel que $\abs{u_n}<\varepsilon$ \ee \begin{comment} \be \item On a $\forall n\in\N$, $n^3+6n^2-5n-2=n(n-1)(n-2)+9n(n-1)+2n-2$\\ D'après la décomposition précédente, pour tout entier naturel $N\geq3$ : \begin{align*} \sum_{n=0}^{N}u_n &=\sum_{n=0}^{N}\frac{n(n-1)(n-2)}{n!}+\sum_{n=0}^{N}\frac{9n(n-1)}{n!}+\sum_{n=0}^{N}\frac{2n}{n!}+\sum_{n=0}^{N}\frac{-2}{n!}\\ &=\sum_{n=3}^{N}\frac{1}{n!}+9\sum_{n=2}^{N}\frac{1}{n!}+2\sum_{n=1}^{N}\frac{1}{n!}-2\sum_{n=0}^{N}\frac{1}{n!}\\ &=\sum_{n=0}^{N-3}\frac{1}{n!}+9\sum_{n=0}^{N-2}\frac{1}{n!}+2\sum_{n=0}^{N-1}\frac{1}{n!}-2\sum_{n=0}^{N}\frac{1}{n!} \end{align*} On reconnaît une somme de séries exponentielles, donc la série converge et \fbox{$\sum_{n=0}^{+\infty}u_n=10e$} \item Comme la série de terme général $u_n$ converge, on a bien \fbox{$\lim_{n\rightarrow +\infty}u_n=0$} On propose le script \begin{verbatim} eps=input("Choisir la valeur de epsilon"); n=0; u=-2; fac=1; while abs(u)>=eps n=n+1; fac=fac*n; u=(n*n*n+6*n*n*-5*n-2)/fac; end; disp(n) \end{verbatim} \ee \end{comment} \newpage \exoe Pour tout $n\in \Nstar$, on pose : \\ \[{A}_{n}=\int_0^{+\infty} \frac{1}{(1+x^{3})^n}\dx\;\;\mbox{et}\;\; v_{n}=\ln{(n^{1/3}A_n)}.\] \\ \be \item {\bf Question de cours} : Donner le développement limité de $\ln{(1+x)}$ à l'ordre 2 au voisinage de $0$.\\ \item Montrer que les suites $(A_n)$ et $(v_n)$ sont bien définies.\\ \item Soit $n\in\Nstar$ \be \item Justifier que $A_{n}=A_{n+1}+\int_0^{+\infty} \frac{x^3}{(1+x^3)^{n+1}}\dx.$ \\ \item À l'aide d'une intégration par parties, exprimer $A_{n+1}$ en fonction de $A_{n}$. \ee \item \be \item Déterminer trois réels $a, b, c$ tels que lorsque $n$ tend vers $+\infty$ : $v_{n+1}-v_n=a+\frac{b}{n}+\frac{c}{n^2}+\circ\pa{\frac{1}{n^2}}$.\\ \item \'Etudier la convergence de la suite $(v_n)$.\\ \ee \item La série $\sum_{n=1}^{+\infty}A_{n}$ converge-t-elle ? \item On pose $b_k=\frac{A_k}{A_1}$. \'Ecrire un programme SCILAB qui pour une valeur de $A$ réelle donnée par l'utilisateur renvoie une valeur de $N$ pour laquelle on a : $$\forall n\geq N,\;\sum_{k=1}^{n}b_k\geq A.$$ \ee \begin{comment} \be \item Programme officiel ECE2 page 9. \fbox{$\ln{(1+x)}=x-\frac{x^2}{2}+\circ (x^2)$} \item La fonction $x\mapsto\frac{1}{(1+x^3)^n}$ est continue sur $\Rplus$. En $+\infty$, comparaison avec une intégrale de Riemann : $\forall x\in \Rplusstar, \frac{1}{(1+x^3)^n}\leq \frac{1}{x^{3n}} $ et $\int_{1}^{+\infty}\frac{1}{x^{3n}}\dx$ converge. De plus, par positivité de l'intégrale, $\forall n\in\Nstar$, $A_n>0$ \fbox{Donc $A_n$ et $v_n$ sont bien définis pour tout $n \in \Nstar$.} \item \be \item $\forall n \in \Nstar, A_{n}=\int_0^\infty \frac{1}{(1+x^{3})^n}\dx=\int_0^\infty \frac{1+x^{3}}{(1+x^{3})^{n+1}}\dx $\\ $A_{n}=\int_0^\infty \frac{1}{(1+x^{3})^{n+1}}\dx+\int_0^\infty \frac{x^3}{(1+x^{3})^{n+1}}\dx$\\ \fbox{$A_{n}=A_{n+1}+\int_0^\infty \frac{x^3}{(1+x^{3})^{n+1}}\dx$} \item $A_{n}=A_{n+1}+\int_0^\infty \frac{x \times x^{2}}{(1+x^{3})^{n+1}}\dx$\\ Par intégration par parties : $A_{n}=A_{n+1}+\crocint{x\times \frac{-1}{3n}\times \frac{1}{(1+x^3)^{n+1}}}_{0}^{+\infty}+\int_{0}^{+\infty}\frac{1}{3n}\times \frac{1}{(1+x^3)^n}\dx $\\ $A_n=A_{n+1}+\frac{1}{3n}A_{n}$. Ainsi \fbox{$A_{n+1}=\frac{3n-1}{3n}A_{n}$}\\ \ee \item \be \item $\forall n \geq 1, v_{n+1}-v_{n}=\ln{((n+1)^{1/3}A_{n+1})}-\ln{(n^{1/3}A_{n})}\\ v_{n+1}-v_{n}=\ln{\pa{\paf{n+1}{n}^{1/3}}}+\ln{\paf{A_{n+1}}{A_n}}\\ v_{n+1}-v_{n}=\frac{1}{3}\ln{\pa{1+\frac{1}{n}}}+\ln{\pa{1-\frac{1}{3n}}}\\ v_{n+1}-v_{n}=\frac{1}{3}\pa{\frac{1}{n}-\frac{1}{2n^2}}+\pa{-\frac{1}{3n}-\frac{1}{18n^2}}+\circ\pa{\frac{1}{n^2}} v_{n+1}-v_{n}=-\frac{2}{9n^2}+\circ\pa{\frac{1}{n^2}}$\\ \fbox{$a=0, b=0, c=-\frac{2}{9}$ conviennent.} \item Par comparaison (les termes sont de signe constant), la série de terme général $v_{n+1}-v_{n}$ converge. Cette série est téléscopique : Or, en posant pour $n\in\Nstar$, $S_n=\sum_{k=1}^{n}(v_{k+1}-v_k)$, on a pour tout $n\in\Nstar$, $S_n=v_{n+1}-v_1$ Comme $(S_n)$ converge vers une limite $\ell$, $(v_{n+1})$ converge vers $\ell+v_1$, et donc \fbox{La suite $(v_n)$ converge} \ee \item Comme pour tout $n\in\Nstar$, $v_n=\ln{\pa{n^{1/3}A_n}}$. Si on note $L$ la limite de la suite $(v_n) $ en $+\infty$, alors $\lim_{n\to+\infty}n^{1/3}A_n ={\e^{L}}$. Donc $A_n\sim\frac{\e^L}{n^{1/3}}$ Or $\sum_{n=1}^{+\infty}\frac{1}{n^{1/3}}$ diverge car $\frac{1}{3}<1$ et les séries sont à termes positifs, donc \fbox{$\sum_{n=1}^{+\infty}A_n$ diverge.} \item On pose $\frac{A_{k+1}}{A_1}=\frac{3k-1}{3k}\times \frac{A_k}{A_1}$, donc $b_{k+1}=\frac{3k-1}{3k}b_k$ On cherche donc à trouver $N$ tel que $\sum_{k=1}^{N}b_k\geq N$. (comme les $b_k$ sont positifs, l'inégalité sera vraie pour $n\geq N$) \begin{verbatim} A=input("Donnez la valeur de A"); b=1; k=1; S=1; while S 0 \\ 0 & \mbox{si }x\leq 0 \end{array} \right.}$$ \'Etablir que $g$ définit bien une densité de probabilité d'une variable aléatoire $Z$. \begin{comment} \bi \item[*] Comme $f$ est nulle sur $\Rmoins$ alors $F\left( x\right) =\int_{-\infty }^{x}f=0$ sur $\Rmoins$ et $1-F\left( x\right) >0$ sur $\Rmoins$. D'autre part, $F$ est d\'{e}rivable l\`{a} o\`{u} la densit\'{e} $f$ est continue. $F$ est d\'{e}rivable sur $\Rplus$ et $F^{\prime }=f>0.$ $F$ est donc strictement croissante sur $\Rplus.$ De plus $% \lim_{+\infty }F=1$ alors $F<1$ sur $\Rplus$ d'o\`{u} $1-F\left( x\right) >0$ sur $\Rplus$. Ainsi, pour tout réel $x, 1-F\left( x\right) >0 $ et $g$ est bien définie \item[*] $g$ est nulle, donc positive sur $\Rmoinsstar$. Sur $\Rplus:$ $0<1-F\left( x\right) \leq 1$ (car $F\left( x\right) $ est une probabilit\'{e} donc inf\'{e}rieure ou \'{e}gale \`{a} $1)$ Donc $\ln \left( 1-F\left( x\right) \right) \leq 0$ et $g\left( x\right) =-f\left( x\right) \ln \left( 1-F\left( x\right) \right) \geq 0$ Ainsi, $g$ est positive ou nulle sur $\R$. \item[*] $F$ est continue sur $\R$ et $1-F\left( x\right) >0$ donc $x\rightarrow \ln \left( 1-F\left( x\right) \right) $ est continue sur $\R$. $f$ est continue sur $\left] -\infty ,0\right[ $ et sur $\left[ 0,+\infty\right[ $ Donc $g$ est continue sur sur $\left] -\infty ,0\right[ $ et sur $\left[ 0,+\infty \right[ $ comme produit de fonctions continues. \item[*] Pour tout réel $M\geq0$: \begin{equation*} \int_{0}^{M}g\left( x\right)\dx=\int_{0}^{M}-f\left( x\right) \ln \left( 1-F\left( x\right) \right)\dx \end{equation*}% Soit $u^{\prime }\left( x\right) =-f\left( x\right) :u\left( x\right) =1-F\left( x\right) :v\left( x\right) =\ln \left( 1-F\left( x\right) \right) :v^{\prime }\left( x\right) =\dfrac{-f\left( x\right) }{1-F\left( x\right) }$ et $u$ et $v$ sont de classe $C^{1}$ sur $\Rplus$ ($1-F\left( x\right) >0$ ) donc \begin{eqnarray*} \int_{0}^{M}g\left( x\right) \dx &=&\left[ \left( 1-F\left( x\right) \right) \ln \left( 1-F\left( x\right) \right) \right] _{0}^{M}-\int_{0}^{M}\left( 1-F\left( x\right) \right) \dfrac{-f\left( x\right) }{1-F\left( x\right) }\dx \\ &=&\left( 1-F\left( M\right) \right) \ln \left( 1-F\left( M\right) \right) -\left( 1-F\left( 0\right) \right) \ln \left( 1-F\left( 0\right) \right) +\int_{0}^{M}f\left( x\right) \dx \end{eqnarray*}% Or, $F\left( 0\right) =\int_{-\infty }^{0}0\dx=0$ donc $\ln \left( 1-F\left( 0\right) \right) =0$ $F\left( M\right) \rightarrow 1$ quand $M\rightarrow +\infty $ et comme $% x\ln \left( x\right) \rightarrow 0$ quand $x\rightarrow 0$ alors $\left( 1-F\left( M\right) \right) \ln \left( 1-F\left( M\right) \right) \rightarrow 0$ Enfin, $\int_{0}^{M}f\left( x\right) \dx\rightarrow \int_{0}^{+\infty }f\left( x\right) \dx=1$ donc \fbox{$\int_{0}^{+\infty }g\left( x\right) \dx$ converge et vaut $1$.} \ei \fbox{$g$ définit bien une densité de probabilité} \end{comment} \newpage \exoe Pour tout entier $n$ non nul, on pose $S_n=\sum\limits_{k=1}^{n}\frac{(-1)^k}{k}$ et $v_n=\frac{1}{n}-\ln\left(1+\frac{1}{n}\right)$. \begin{enumerate} \item%1 {\bf Question de cours:} \'Enoncer les critères de comparaison des séries. \item %2 \begin{enumerate} \item Montrer que $v_n\underset{n\to+\infty}{\sim}\frac{1}{2n^2}$. \item En déduire que la série $\sum\limits_{n\geqslant 1}v_n$ converge, puis que la suite $\left(\sum\limits_{k=1}^{n}\frac{1}{k}-\ln(n)\right)_{n\geqslant 1}$ converge. On notera $\gamma$ sa limite. \end{enumerate} \item %3 \begin{enumerate} \item Montrer que la suite $(S_{2n})_{n\geqslant 1}$ est décroissante et que la suite $(S_{2n+1})_{n\geqslant 0}$ est croissante. \item En déduire que la série $\sum\limits_{n\geqslant 1}\frac{(-1)^n}{n}$ converge. \end{enumerate} \item %4 \begin{enumerate} \item Montrer que pour tout entier $n$ non nul, $\sum\limits_{k=1}^{2n}\frac{1}{k}=\ln(2)+\ln(n)+\gamma+\underset{n\to +\infty}{o}(1)$. % Si formulation trop compliquée on passe à une formulation avec des limites. \item Montrer que pour tout entier naturel $n$ non nul, $\sum\limits_{k=1}^{2n}\frac{(-1)^k}{k}=-\sum\limits_{k=1}^{2n}\frac{1}{k}+2\sum\limits_{k=1}^{n}\frac{1}{2k}$. \item En déduire la valeur de $\sum\limits_{n=1}^{+\infty} \frac{(-1)^n}{n}$. \end{enumerate}\end{enumerate} \begin{comment} \begin{enumerate} \item Programme officiel ECE2 page 8. Soient $\sum\limits_{n\geqslant 0} u_n$ et $\sum\limits_{n\geqslant 0} v_n$ deux séries à termes positifs.\\ Si $\forall n\in\N,\: 0\leqslant u_n\leqslant v_n$ et que $\sum\limits_{n\geqslant 0} v_n$ converge, alors $\sum\limits_{n\geqslant 0}u_n$ converge. \\ Si $ u_n=\underset{n\to +\infty} o\left( v_n\right)$ et que $\sum\limits_{n\geqslant 0} v_n$ converge, alors $\sum\limits_{n\geqslant 0}u_n$ converge. \\ Si $ u_n\underset{n\to +\infty}{\sim} v_n$, alors $\sum\limits_{n\geqslant 0} v_n$ et $\sum\limits_{n\geqslant 0}u_n$ sont de même nature. \item %2 \begin{enumerate} \item %a Rappelons que $\ln\left(1+\frac{1}{n}\right)=\frac{1}{n}-\frac{1}{2n^2}+\underset{n\to +\infty}{o}\left(\frac{1}{n^2}\right)$.\\ Donc $v_n=\frac{1}{2n^2}+\underset{n\to +\infty}{o}\left(\frac{1}{n^2}\right)$.\\ Ainsi \fbox{$v_n\underset{n\to +\infty}{\sim} \frac{1}{2n^2}$}. \item %b La série de Riemann $\sum\limits_{n\geqslant 1} \frac{1}{n^2}$ converge (car $2>1$). Donc par critère d'équivalence, $\sum\limits_{n\geqslant 1}v_n$ converge.\\ Par définition de la convergence d'une série, la suite $\left(\sum\limits_{k=1}^nv_k\right)_{n\geqslant 1}$ converge.\\ Or $\forall n\in\N^*,\: \sum\limits_{k=1}^nv_k=\sum\limits_{k=1}^n\left(\frac{1}{k}-\ln\left(1+\frac{1}{k}\right)\right)=\sum\limits_{k=1}^n\left(\frac{1}{k}-(\ln(k+1)-\ln(k))\right)=\sum\limits_{k=1}^n\frac{1}{k}-\ln(n+1)$.\\ Or, $\sum\limits_{k=1}^n\frac{1}{k}-\ln n=\sum\limits_{k=1}^n\frac{1}{k}-\ln(n+1)+\ln\paf{n+1}{n}$ et $\lim\limits_{n\to+\infty}\ln\paf{n+1}{n}=0$ Donc \fbox{la suite $\left(\sum\limits_{k=1}^n\frac{1}{k}-\ln n\right)_{n\geqslant 1}$ converge.} \end{enumerate} \item %3 \begin{enumerate} \item %a \begin{description} \item[$\diamond$] Pour tout entier naturel $n$ non nul, \begin{eqnarray*} S_{2(n+1)}-S_{2n} & = & \sum\limits_{k=1}^{2n+2}\frac{(-1)^k}{k} -\sum\limits_{k=1}^{2n}\frac{(-1)^k}{k}\\ & = & \frac{1}{2n+2}-\frac{1}{2n+1} \end{eqnarray*} Ainsi $S_{2(n+1)}-S_{2n}\leqslant 0$.\\ \fbox{Donc la suite $(S_{2n})_{n\geqslant 1}$ est décroissante.} \item[$\diamond$]Pour tout entier naturel $n$, \begin{eqnarray*} S_{2(n+1)+1}-S_{2n+1} & = & \sum\limits_{k=1}^{2n+3}\frac{(-1)^k}{k} -\sum\limits_{k=1}^{2n+1}\frac{(-1)^k}{k}\\ & = & \frac{1}{2n+2}-\frac{1}{2n+3} \end{eqnarray*} Ainsi $S_{2(n+1)+1}-S_{2n+1}\geqslant 0$.\\ Donc\fbox{ la suite $(S_{2n+1})_{n\geqslant 0}$ est croissante.} \end{description} \item%b \begin{description} \item[$\bullet$] Remarquons que $\forall n\in\N^*,\: S_{2n+1}-S_{2n}=-\ \frac{1}{2n+1}$.\\ Alors $\lim\limits_{n\to+\infty}S_{2n+1}-S_{2n}=0$. \item[$\bullet$] Les suites $(S_{2n})_{n\geqslant 1}$ et $(S_{2n+1})_{n\geqslant 0}$ sont monotones et de monotonies différentes \end{description} Donc les suites $(S_{2n})_{n\geqslant 1}$ et $(S_{2n+1})_{n\geqslant 0}$ sont adjacentes. Donc les suites $(S_{2n})_{n\geqslant 1}$ et $(S_{2n+1})_{n\geqslant 0}$ convergent vers une même limite.\\ Alors la suite $(S_n)_{n\geqslant 1}$ converge. Donc par définition \fbox{la série $\sum\limits_{n\geqslant 1}\frac{(-1)^n}{n}$ converge. } \end{enumerate} \item%4 \begin{enumerate} \item%a D'après la question 2(b), la suite $\left(\sum\limits_{k=1}^{2n}\frac{1}{k}-\ln(2n)\right)_{n\geqslant 1}$ converge vers $\gamma$.\\ Donc $\sum\limits_{k=1}^{2n}\frac{1}{k}=\ln(2 n)+\gamma +\underset{n\to+\infty}{o}(1)$.\\ ou encore \fbox{$\sum\limits_{k=1}^{2n}\frac{1}{k}=\ln(2)+\ln (n)+\gamma +\underset{n\to+\infty}{o}(1)$.} \item %b Soit $n$ un entier naturel non nul. \begin{eqnarray*} \sum\limits_{k=1}^{2n}\frac{(-1)^k}{k}& = & \sum\limits_{k=1\atop k\textrm{ pair}}^{2n}\frac{(-1)^k}{k}+\sum\limits_{k=1\atop k\textrm{ impair}}^{2n}\frac{(-1)^k}{k} \\ & = & \sum\limits_{k=1}^{n}\frac{1}{2k}+\sum\limits_{k=0}^{n-1}\frac{-1}{2k+1}\\ &= & 2\sum\limits_{k=1}^{n}\frac{1}{2k}-\sum\limits_{k=1}^{n}\frac{1}{2k}-\sum\limits_{k=0}^{n-1}\frac{1}{2k+1}\\ & = & 2\sum\limits_{k=1}^{n}\frac{1}{2k}-\sum\limits_{k=1}^{2n}\frac{1}{k} \end{eqnarray*} \item%c Pour tout entier $n$ non nul, $2\sum\limits_{k=1}^{n}\frac{1}{2k}-\sum\limits_{k=1}^{2n}\frac{1}{k}=\sum\limits_{k=1}^{n}\frac{1}{k}-\sum\limits_{k=1}^{2n}\frac{1}{k}$.\\ Alors $\sum\limits_{k=1}^{2n}\frac{(-1)^k}{k}=\ln(n)+\gamma-\ln(2)-\ln(n)-\gamma+\underset{n\to+\infty}{o}\left(1\right)$. Donc $\sum\limits_{k=1}^{2n}\frac{(-1)^k}{k}=-\ln(2)+\underset{n\to+\infty}{o}\left(1\right)$.\\ Ainsi, puisque cette série converge $\sum\limits_{n=1}^{+\infty}\frac{(-1)^n}{n}=\lim\limits_{n\to +\infty} \sum\limits_{k=1}^{2n} \frac{(-1)^k}{k}$\fbox{$=-\ln 2$. } \end{enumerate} \end{enumerate} \end{comment} \newpage \Espe Soient $p\in ]0,1[$ et la fonction SCILAB {\tt X} suivante \begin{verbatim} function x=X(p) k=1; y=0; a=rand(); while a>p y=y+1; k=k+1; a=rand(); end; while a<=p k=k+1; a=rand() end; x=k-1-y; endfunction \end{verbatim} En s'aidant d'un lancer d'une pi\`ece dont la probabilit\'e d'obtenir "face" est $p$. \be \item Interpreter ce que simule la fonction {\tt X}? \item Soit $X$ la variable al\'eatoire simulée par la fonction ci-dessus. \bi \item Quelle est la loi de $X$? \item Quelle est l'esp\'erance de $X$ si elle existe? \ei \ee \begin{comment} \be\item La fonction d\'etermine \fbox{ la longueur de la premi\`ere s\'equence de "face".} {\tt y } est le nombre de "pile" pr\'ec\'edant la premi\`ere s\'equence de "face". \item Soit $q=1-p$ Posons $X=0$ si "face" n'appara\^\i t jamais. $X(\Omega) = \N$ Soit $k \in \N^*$ $[X=k] = \displaystyle\bigcup_{y=0}^{+\infty} \Big( B_y \cap A_{y+1,k} \Big)$ \quad (union disjointe). o\`u $B_y$ est l' \'ev\'enement: " $y$ piles sont apparus avant la premi\`ere s\'equence de faces " $ A_{y+1,k}$ est l'\'ev\'enement : " face est apparu aux rangs $y+1, ..., y+k$ et pile au rang $y+k+1$." $\P(X=kà=\displaystyle \sum_{y=0}^{+\infty} q^y p^k q$ \quad les jets de pi\`eces \'etant suppos\'es ind\'ependants. $\P(X=k) = \displaystyle{q p^k \over 1-q} = qp^{k-1}$ \fbox{$X$ suit donc la loi g\'eom\'etrique de param\`etre $q$} Ce r\'esultat \'etait pr\'evisible: $X$ et le rang du premier "pile" \`a partir de l'obtention du premier "face" et la loi géométrique est "sans mémoire" \medskip Donc \fbox{$\E(X)=\displaystyle{1 \over q}$} \ee \end{comment} \newpage \exoe Soient $n$ un entier naturel supérieur ou égal à $2$ et $k$ un entier naturel inférieur ou égal à $n.$ \\ Sur le segment [$0,1]$, on place au hasard $n$ points, on les ordonne dans l'ordre croissant et, pour $n$ fixé, on appelle $X_{k}$ la variable aléatoire égale à l'abscisse du $k$-ième point. On définit pour $x\in]0,1[$, $Y_x$ le nombre de points qui ont été placés dans le segment $[0;x]$ \be\item {\bf Question de cours}: L'inégalité de Bienaymé-Tchebychev. \item Déterminer la loi de $Y_x$ pour $x\in]0;1[$. \item Soit $(p, q) \in \N^{2}$. On pose $I_{p, q}=\int_{0}^{1} x^{p}(1-x)^{q} d x$. Montrer que: $$ \forall p \in \Nstar, \quad \forall q \in \N; \quad I_{p, q}=\frac{p}{q+1} I_{p-1, q+1} $$ En déduire l'expression de $I_{p, q}$. \item Soit $k\in[|1;n|]$ fixé. \be \item Déterminer la fonction de répartition de $X_k$ (on pourra laisser l'expression sous la forme d'une somme.) \item Montrer que $X_{k}$ est une variable à densité dont une densité $f$ est donnée par: $$ \forall x \in \R \quad f(x)=\left\{\begin{array}{cc} k\left(\begin{array}{c} n \\ k \end{array}\right) x^{k-1}(1-x)^{n-k} & \text { si } \quad 00, \quad\big[\left|Z_n\right|>\varepsilon\big] \subset\big[(\left|Z_n-\E\left(Z_n\right)\right|+\left|\E\left(Z_n\right)\right|)>\varepsilon\big] $$ Soit $\varepsilon>0$ fixé. On admet que, comme $\lim _{n \rightarrow \infty}\E(Z_n)=0$, pour $n$ assez grand $\abs{\E(Z_n)}<\frac{\varepsilon}{2}$ et donc $\varepsilon-\abs{\E(Z_n)}>\frac{\varepsilon}{2}$ \item Montrer que pour $n$ assez grand, $\P\left(\abs{ Z_{n} }>\varepsilon\right)\leq \frac{4V(Z_n)}{\varepsilon^2}$. \ee\ee \begin{comment} \be\item programme officiel ECE2 page 17. Si X est une variable aléatoire admettant une espérance et une variance : $\forall \varepsilon>0,\quad\P(\abs{X-\E(X)}\geq \varepsilon)\leq \frac{V(X)}{\varepsilon^2}$ \item \fbox{ $Y_x$ une binomiale de paramètres $n$ et $x$} \item On obtient l'égalité $I_{p, q}=\frac{p}{q+1} I_{p-1, q+1}$ à l'aide d'une intégration par parties. Par itérations : $ I_{p, q}=\frac{p(p-1) \ldots 1}{(q+1)(q+2) \ldots(q+p)} I_{0, p+q}=\frac{p ! q !}{(p+q) !} \int_{0}^{1}(1-x)^{p+q} \dx$ \fbox{$I_{p, q}=\frac{1}{(p+q+1)\left(\begin{array}{c} p+q \\ p \end{array}\right)} $} \item\be\item Si l'on note $F_k$ la fonction de répartition $F_k(x)=0$ si $x\leq 0$ et $F_k(x)=1$ si $x\geq 1$. Pour $x\in]0;1[$, $F_k(x)=\P(X_k\leq x)=\P(Y_x\geq k)$ \fbox{$\forall x\in]0;1[$, $F_{k}(x)=\sum_{i=k}^{n}\left(\begin{array}{c} n \\ i \end{array}\right) x^{i}(1-x)^{n-i} $} \item $F_{k}$ est continue sur $\R$ et est de classe $C^{1}$ sur $\R\backslash\{0,1\}$ donc $X_{k}$ est à densité et une densité est donnée par : * $f_{k}(x)=0$ si $x<0$ ou $x>1$ *$ \text { si } 0\varepsilon$, alors $\left|Z_{n}-E\left(Z_{n}\right)\right|+\left|E\left(Z_{n}\right)\right|>\varepsilon$ On a bien l'inclusion demandée \fbox{$\left[\left|Z_{n}\right|>\varepsilon\right] \subset\left[\left|Z_{n}-E\left(Z_{n}\right)\right|+\left|E\left(Z_{n}\right)\right|>\varepsilon\right]$} \item On obtient donc $\P\left(\left|Z_{n}\right|>\varepsilon\right) \leq \P\left(\left|Z_{n}-E\left(Z_{n}\right)\right|>\varepsilon-\left|E\left(Z_{n}\right)\right|\right)$ Mais pour $n$ assez grand $\abs{\E(Z_n)}<\frac{\varepsilon}{2}$ et donc $\varepsilon-\left|E\left(Z_{n}\right)\right|>\frac{\varepsilon}{2}$ $ \P\left(\left|Z_{n}\right|>\varepsilon\right) \leq \P\left(\left|Z_{n}-\E\left(Z_{n}\right)\right|>\E(Z_n)-\frac{\varepsilon}{2}\right)\leq \P\left(\left|Z_{n}-\E\left(Z_{n}\right)\right|>\frac{\varepsilon}{2}\right)$ Or d'après Bienaymé-Tchébytchev $\P\left(\left|Z_{n}-\E\left(Z_{n}\right)\right|>\frac{\varepsilon}{2}\right) \leq \frac{4 V\left(Z_{n}\right)}{\varepsilon^{2}}.$ Ains i \fbox{$\P\left(\left|Z_{n}\right|>\varepsilon\right)\leq \frac{4 V\left(Z_{n}\right)}{\varepsilon^{2}}$} \ee\ee \end{comment} \newpage \Espe Soit $E$ un $\R$ espace vectoriel de dimension 3.\\ On note $Id_E$ et $O_{{\mathcal L}(E)}$ les applications identité de $E$ dans $E$, et constante nulle de $E$ dans $E$.\\ On considère un endomorphisme $f$ de $E$ tel que : $$f\neq O_{{\mathcal L}(E)},\;\;\; f^2+Id_E\neq O_{{\mathcal L}(E)}\;\;{\mbox et }\;\;f\circ(f^2+Id_E)=O_{{\mathcal L}(E)}$$ où $f^2$ désigne $f\circ f$. \be \item Déterminer sp($f$). L'application $f$ est-elle diagonalisable ? \item L'application $f^2+Id_E$ est-elle bijective ? \ee \begin{comment} \be\item Le polynôme $X(X^2+1)$ est un polynôme annulateur de $f$ et son unique racine est 0. $0$ est donc la seule valeur propre possible de $f$. $f^2+Id_E\neq O_{{\mathcal L}(E)}$ donc il existe un vecteur $x_0 \in E$ tel que $\pa{f^2+Id_E}(x_0)\neq0$. \\ $f\circ(f^2+Id_E)=O_{{\mathcal L}(E)}$ donc $f\pa{(f^2+Id_E)(x_0)}=0$. Le vecteur $(f^2+Id_E)(x_0)$ est donc un vecteur non nul du noyau de $f$ Ainsi, \fbox{Sp($f)=\acco{0}$.}\\ Par l'absurde, si $f$ était diagonalisable, elle serait représentée dans une base de vecteurs propres par la matrice nulle, donc $f$ serait nulle, ce qui n'est pas le cas. \fbox{ Donc $f$ n'est pas diagonalisable.} \item Si $f^2+Id_E$ était bijective, on pourrait composer à droite par son inverse dans la relation $ f\circ(f^2+Id_E)=O_{{\mathcal L}(E)}$ et obtenir que $f=O_{{\mathcal L}(E)}$, ce qui est absurde. \fbox{ $f^2+Id_E$ n'est pas bijectif.} \ee \end{comment} \newpage \exoe \noindent Soit $\mathcal{S}$ le $\R$-espace vectoriel des suites r\'eelles. Soit $\mathcal{C}$ l'ensemble des suites r\'eelles $(a_n)_{n\in \N}$ telles que la s\'erie $\sum a_n$ converge. \begin{enumerate} \item Question de cours.\\ Rappeler les conditions de convergence des séries géométriques, géométriques dérivées et des séries exponentielles. En cas de convergence, rappeler la valeur de la somme de ses séries.\\ \item \begin{enumerate} \item La suite $A_1=\left(\dfrac{1}{n!}\right)$ appartient-elle \`a $\mathcal{C}$ ? \item La suite $A_2=((-1)^n)$ appartient-elle \`a $\mathcal{C}$ ? \item Montrer que $\mathcal{C}$ est un sous-espace vectoriel de $\mathcal{S}$. \end{enumerate} \bigskip \bigskip Pour toute suite $A=(a_n)\in \mathcal{S}$, on dit que la suite $A=(a_n)$ v\'erifie la propri\'et\'e $\mathcal{P}$ si \begin{description} \item (i) pour tout $x\in ]-1,1[$, la s\'erie $\sum a_nx^n$ converge. \item (ii) $f:x\mapsto \displaystyle \sum_{n=0}^{+\infty} a_nx^n$ admet une limite finie lorsque $x$ se rapproche de $1$ par valeurs inf\'erieures.\\ On note alors $$\ell(A)=\displaystyle \lim_{x\to 1-} \left(\sum_{n=0}^{+\infty} a_nx^n\right) \in \R.$$ \end{description} \item \begin{enumerate} \item La suite $A_1=\left(\dfrac{1}{n!}\right)$ v\'erifie-t-elle la propri\'et\'e $\mathcal{P}$ ? Si oui, d\'eterminer $\ell(A_1)$. \item La suite $A_2=((-1)^n)$ v\'erifie-t-elle la propri\'et\'e $\mathcal{P}$ ? Si oui, d\'eterminer $\ell(A_2)$. \item Soit $A_3=(n^2)$ v\'erifie-t-elle la propri\'et\'e $\mathcal{P}$ ? Si oui, d\'eterminer $\ell(A_3)$. \end{enumerate} \item Soit $A=(a_n)$ une suite de $\mathcal{S}$. On suppose que pour tout $n\in \N$, $a_n\geq 0$ et que la suite v\'erifie la propri\'et\'e $\mathcal{P}$. \be\item Montrer que $\forall n\in\N$, $\sum_{k=0}^{n}a_k\leq \ell(A)$. \item En déduire que la suite $A$ appartient \`a $\mathcal{C}$. \ee \item Soit $A=(a_n)_{n\in \N}$ une suite de r\'eels telle que $$\forall \, n \in \N, \qquad a_{n+3}=a_n.$$ \begin{enumerate} \item Donner une condition n\'ecessaire et suffisante pour que $A$ soit dans $\mathcal{C}$. \item Montrer que la suite $A$ v\'erifie la propri\'et\'e $\mathcal{P}$ si et seulement si $\displaystyle a_0+a_1+a_2=0.$ \end{enumerate} \end{enumerate} \begin{comment} \begin{enumerate} \item Programme officiel ECE1, page 16. Les séries $\sum q^n$, $\sum nq^{n-1}$ et $\sum n(n-1)q^{n-2}$ convergent ssi $\abs{q}<1$ et dans ce cas : \fbox{ $\sum_{n=0}^{+\infty}q^n=\frac{1}{1-q}$, $\sum_{n=1}^{+\infty}nq^{n-1}=\frac{1}{(1-q)^2}$ et $\sum_{n=1}^{+\infty}n(n-1)q^{n-2}=\frac{2}{(1-q)^3}$} Pour tout $x\in\R$, la série $\sum \frac{x^n}{n!}$ converge et \fbox{ $\sum_{n=0}^{+\infty}\frac{x^n}{n!}=e^x$} \item \begin{enumerate} \item Comme la s\'erie $\displaystyle \sum \dfrac{1}{n!}$ converge (en tant que s\'erie exponentielle), on conclut que: la suite \fbox{$\left(\dfrac{1}{n!}\right)$ appartient \`a $\mathcal{C}$.} \item La s\'erie $\sum (-1)^n$ \fbox{diverge grossi\`erement}, donc : la suite $((-1)^n)$ n'appartient pas \`a $\mathcal{C}$. \item On note que $\mathcal{C} \subset \mathcal{S}$. L'ensemble $\mathcal{C}$ est non vide car il contient la suite nulle et l'ensemble $\mathcal{C}$ est stable par combinaisons lin\'eaires. On conclut ainsi que \fbox{$\mathcal{C}$ est un sous-espace vectoriel de $\mathcal{S}$.} \end{enumerate} \item \begin{enumerate} \item Pour tout $x\in ]-1,1[$, la s\'erie $\displaystyle \sum \dfrac{x^n}{n!}$ converge et a pour somme $\e^x$. Par suite, $\displaystyle \lim_{x\to 1^-} \left(\sum_{n=0}^{+\infty} \dfrac{x^n}{n!}\right)=\e^1 \in \R$. Donc, la suite $A_1=\left(\dfrac{1}{n!}\right)$ v\'erifie la propri\'et\'e $\mathcal{P}$, avec \fbox{$\ell(A_1)=\e$.} \item La s\'erie g\'eom\'etrique $\displaystyle \sum (-x)^n$ converge si et seulement si $|-x|<1$, donc si et seulement si $x\in ]-1,1[$. De plus, pour tout $x\in ]-1,1[$, $\displaystyle \sum_{n=0}^{+\infty}(-x)^n=\dfrac{1}{1+x}$ qui a pour limite $\dfrac{1}{2}$ quand $x$ tend vers $1^-$. On conclut que la suite $A_2=((-1)^n)$ v\'erifie la propri\'et\'e $\mathcal{P}$ avec \fbox{$\ell(A_2)=\dfrac{1}{2}$.} \item Pour tout $x\in ]-1,1[$ et pour tout $n\in \N$, on \'ecrit $n^2x^n=x^2\,n(n-1)x^{n-2}+x\, n \, x^{n-1}.$ La s\'erie $\displaystyle \sum n^2x^n$ converge comme combinaison lin\'eaire de deux s\'eries g\'eom\'etriques d\'eriv\'ees convergentes et $$\sum_{n=0}^{+\infty} n^2x^n=x^2\,\sum_{n=2}^{+\infty}n(n-1)x^{n-2}+x\,\sum_{n=1}^{+\infty} n \, x^{n-1}=\dfrac{2x^2}{(1-x)^3}+\dfrac{x}{(1-x)^2}=\dfrac{x(x+1)}{(1-x)^3}.$$ Cette quantit\'e n'a pas de limite finie quand $x$ tend vers $1^-$. On conclut que la suite \fbox{$A_3$ ne v\'erifie pas la propri\'et\'e $\mathcal{P}$.} \end{enumerate} \item \be\item Pour tout $n\in \N$, on pose $S_n=\displaystyle \sum_{k=0}^n a_k$, somme partielle de rang $n$ de la s\'erie $\displaystyle \sum a_k$. Par positivit\'e de la suite $(a_k)$, la suite $(S_n)_{n\in \N}$ est croissante. Montrons qu'elle est major\'ee. Pour tout $n\in \N$ et pour tout $x\in [0,1[$, $\displaystyle \sum_{k=0}^{n} a_kx^k\leq \sum_{k=0}^{+\infty} a_kx^k.$ Comme la suite $A$ v\'erifie la propri\'et\'e $\mathcal{P}$, on peut fair tendre $x$ vers $1^-$ dans cette in\'egalit\'e et l'on obtient : \fbox{$\displaystyle S_n=\sum_{k=0}^n a_k\leq \ell(A)$.} \item La suite $(S_n)$ est croissante et major\'ee, donc elle converge. Ainsi, la s\'erie $\displaystyle \sum a_k$ converge, ce qui prouve que : \fbox{la suite $A$ appartient \`a $\mathcal{C}$.}\ee \item \begin{enumerate} \item Supposons que $A$ soit un \'el\'ement de $\mathcal{C}$. Alors, la s\'erie $\displaystyle \sum a_n$ converge et, par suite, $\displaystyle \lim_{n\to+\infty}(a_n)=0.$ En tant que suite extraite de $(a_n)$, pour tout $i\in \N$, $\displaystyle \lim_{n\to+\infty}(a_{3n+i})=0.$ Or, pour tout $i\in \{0,1,2\}$ et tout $n\in \N$, $a_{3n+i}=a_i$. La suite \fbox{$A$ est donc la suite nulle.} \\ R\'eciproquement, la suite nulle est bien élément de $\mathcal{C}$ et vérifie l'hypothèse. \item $\bullet$ La suite $(|a_n|)$ est born\'ee par le r\'eel $M=\max\{|a_0|, |a_1|, |a_2|\}$. Ainsi, pour tout $n\in \N$ et pour tout $x\in]-1,1[$, $|a_nx^n|\leq M|x|^n$. Par crit\`ere de comparaison des s\'eries \`a termes positifs, la s\'erie $\displaystyle \sum a_nx^n$ converge absolument, donc converge.\\ $\bullet$ Pour tout $x\in ]-1,1[$, pour tout $N\in \N^*$, $$S_N(x)=\sum_{n=0}^{3N-1} a_nx^n=\sum_{i=0}^{N-1} \left(a_{3i}x^{3i}+a_{3i+1}x^{3i+1}+a_{3i+2}x^{3i+2}\right).$$ $$S_N(x)=\sum_{i=0}^{N-1} \left(a_{0}x^{3i}+a_{1}x^{3i+1}+a_{2}x^{3i+2}\right)=(a_0+a_1x+a_2x^2)\sum_{i=0}^{N-1}\left(x^3\right)^i.$$ $$\lim_{N\to +\infty} \left(S_N(x)\right)=\dfrac{a_0+a_1x+a_2x^2}{1-x^3}=\dfrac{a_0+a_1x+a_2x^2}{(1-x)(1+x+x^2)}.$$ Il suit que $x\mapsto\displaystyle \sum_{n=0}^{+\infty} a_nx^n$ admet une limite finie quand $x$ tend vers $1^-$ si et seulement si $1$ est racine de $a_0+a_1x+a_2x^2$, donc \fbox{si et seulement si $a_0+a_1+a_2=0.$} \end{enumerate} \end{enumerate} \end{comment} \newpage \Espe Soit C un réel, $\lambda$ un réel strictement positif et $f$ la fonction définie par: $$ \forall x \in \R \quad f(x)=\left\{\begin{array}{cc} C e^{-\lambda x} & \text { si } \quad x \geq 1 \\ 0 & \text { sinon } \end{array}\right. $$ \be \item Exprimer C en fonction de $\lambda$ pour que $f$ soit une densité de probabilité. Soit alors, pour cette valeur de C, une variable aléatoire X admettant $f$ pour densité. \item Exprimer X simplement à l'aide d'une variable aléatoire suivant une loi usuelle. \item On a obtenu les moyennes de 5 expériences de $10^{5}$ réalisations indépendantes de $X:$ $\begin{array}{lllll}1.5016443 & 1.501896 & 1.4995983 & 1.5003972 & 1.4982147\end{array}$ Que pensez-vous de la valeur de $\lambda$ ? \'Ecrire des lignes de code Scilab qui demande $\lambda$ à l'utilisateur, 5 expériences de $10^{5}$ réalisations indépendantes de $X$, affiche leurs 5 moyennes, puis propose une estimation de $\lambda$. \ee \begin{comment} \be\item $f$ est continue sur $\R\backslash\{1\}, f \geq 0 \Rightarrow C \geq 0$ et $\int_{-\infty}^{+\infty} f=1 \Rightarrow $\fbox{$C=\lambda e^{\lambda}$.} \item Pour $x\geq 1$, $f(x)=\lambda e^{\lambda}.e^{-\lambda x}=\lambda\exp(-(\lambda(x-1))=g(x-1)$ où $g$ est la densité d'une variable aléatoire Y suivant une loi exponentielle de paramètre $\lambda$. Ce résultat reste vrai si $x<1$ Ainsi $\P(X\leq x)=\int_{-\infty}^xf(t)\dt=\int_{-\infty}^{x}g(t-1)\dt=\int_{-\infty}^{x-1}g(t)\dt=\P(Y\leq x-1)=\P(Y+1\leq x)$. On peut écrire \fbox{$X=Y+1$ où Y suit une loi exponentielle de paramètre $\lambda$.} \item Chaque moyenne obtenue est la moyenne empirique des $10^{5}$ réalisations: une valeur de l'espérance de $X$ est environ $1.5$ (on peut invoquer la loi des grands nombres) Or, $\E(X)=1+\frac{1}{\lambda}$, donc \fbox{$\lambda$ est voisin de $\frac{1}{1.5-1}=2$. } On propose \begin{verbatim} lambda= input('lambda='); A=grand(5,100000,"exp",1/lambda); A=1+A; V=mean(A,'c'); disp(V'); disp(1/(mean(V)-1)); \end{verbatim} Explications: {\tt grand(5,100000,"exp",1/lambda)} génère $5 \times 100000$ réalisations indépendantes d'une variable de loi exponentielle de paramètre $\lambda$. {\tt A=1+A} ajoute 1 à chaque élément de $A$. {\tt V=mean(A,'c')} : V est une matrice colonne dont le $i$-ème élément est la moyenne des éléments de la $i$ème ligne de $A$. {\tt V'} transpose $V$. \ee \end{comment} \end{document}